13 Immunology (161) Flashcards

You may prefer our related Brainscape-certified flashcards:
1
Q

Case
A 43-year-old woman presents with a history of itchy eyes, dry mouth, and changes in her facial appearance for the past few months. During the physical exam, the patient shows dry eyes, white patches on the tongue, and enlargement of the parotid glands. There is no clinical evidence of active arthritis.

What antibody is most likely be positive in this patient?

1 Cytoplasmic antineutrophil cytoplasmic autoantibodies (C-ANCA)
2 Anti-DNA topoisomerase I antibodies (Scl-70)
3 Antinuclear antibodies AntiSm
4 Antinuclear antibodies ds-DNA
5 Anti-ribonucleoprotein antibodies SS-A (Ro)

A

Anti-ribonucleoprotein antibodies SS-A (Ro)

Sjögren syndrome is an autoimmune disease, most common in women, manifested usually by dry eyes and dry mouth as result of destruction of the lacrimal and parotid glands, which are enlarged (Mikulicz syndrome). These patients can also have mild arthritis, but sometimes Sjögren syndrome may also be associated with other autoimmune diseases, such as SLE, where the compromise of the joints can be more severe. Anti-ribonucleoprotein antibodies SS-A (Ro) and SS-B (La) are positive. Recent studies show that there are specific kinds of anticentromere antibodies that can be also positive.
Cytoplasmatic antineutrophil cytoplasmatic autoantibodies (C-ANCA) are positive in granulomatosis with polyangiitis.
Anti-DNA topoisomerase I antibodies (Scl-70) are positive in patients with diffuse scleroderma.
Antinuclear antibodies AntiSm and antinuclear antibodies ds-DNA are positive in systemic lupus erythematosus (SLE).

How well did you know this?
1
Not at all
2
3
4
5
Perfectly
2
Q

A 9-year-old boy presents with a rash on his buttocks. The patient states that the rash has been present for the past few days and is accompanied with joint pain in his knees and wrists. Blood work is drawn and is notable for a serum creatinine clearance of 1.3mg/dL. Vital signs are normal. Physical exam demonstrates palpable purpura on the buttocks. There is mild tenderness in the knee and wrist joints. The patient was diagnosed with Henoch-Schonlein purpura. Which of the following is the most likely mechanism of this disease?

1 Type I hypersensitivity
2 Type II hypersensitivity
3 Type III hypersensitivity
4 Type IV hypersensitivity
5 Type V hypersensitivity

A

Type III hypersensitivity

Henoch-Schonlein purpura occurs due to type III hypersensitivity reaction or immune-complex mediated reaction. Henoch-Schonlein purpura is a disease characterized by palpaple purpura, joint pain, gastrointestinal symptoms, and glomerulonephritis. The disease is usually seen in children, and is more common in boys than in girls. The mechanism of Henoch-Schonlein is a type III hypersensitivity also known as immune complex hypersensitivity. Other examples include serum sickness, SLE, arthus reactions, and rheumatoid arthritis.
Type I hypersensitivity is also known as immediate or anaphylactic hypersensitivity.
Type II hypersensitivity is also known as cytotoxic hypersensitivity. The antigens are normally endogenous, although chemicals (i.e. drugs) can also lead to type II hypersensitivity. A classic example is myasthenia gravis.
Type IV is also known as delayed type hypersensitivity and is seen in tuberculosis, and poison ivy (contact dermatitis).
A reaction in which a directly stimulatory autoantibody acts through a target receptor is called Type V hypersensitivity reaction. An example is Graves’ disease in which the antibody against the TSH receptor stimulates the thyroid gland to cause hyperthyroidism.

How well did you know this?
1
Not at all
2
3
4
5
Perfectly
3
Q

You perform a kidney transplant on a 62 year-old male. Unfortunately, the lab mis-typed the organ and your patient begins to reject the graft within a week of receiving it. As your patient’s macrophages and cytotoxic T cells begin to reject the grafted organ, what cells specifically are they rejecting?
Answer Choices

1 Cells of the loop of Henle
2 Capillary endothelial cells
3 Granular juxtaglomerular cells
4 Glomerular capsular cells
5 Cells of the collecting tubules

A

Capillary endothelial cells

The first cells that the host’s immune system encounter are those of the capillaries in which they travel. All of the immune response is directed against these cells. Microthrombi form in the capillary beds of the transplanted organ and it dies from hypoxia.

How well did you know this?
1
Not at all
2
3
4
5
Perfectly
4
Q

A proud mother brings her healthy, six week old daughter in to your office for her first “shots.” As your nurse administers the vaccine, you reflect upon what you know about vaccines and humoral immunity. You realize that this infant is naive to the antigen that is being introduced to her system, so her initial antibody response will be weak. You take comfort in that subsequent introductions of the antigen will produce a more protective immune response. What class of antibodies will be produced by your young patient against the vaccine?

1 IgG1
2 IgG2
3 IgG3
4 IgM
5 IgE

A

IgM

Under normal circumstances, IgM is produced during the initial response to an antigen. Subsequent exposure to the same antigen results in class-switching by the B cells to produce IgG1.

How well did you know this?
1
Not at all
2
3
4
5
Perfectly
5
Q

What characteristics of IgM make it a good antibody for initial exposure to an antigen?

1 It does not fix complement very well, so it would be less likely for host cells to be damaged by complement
2 Its pentameric structure make it ideal for being secreted and binding to mucosal surfaces to protect the body from invading
organisms
3 Its low affinity and high avidity make it less specific but highly effective at binding to invading antigens
4 In its monomeric form, it binds readily to the surface of mast cells, which, in turn, can release inflammatory mediators that will help eliminate the antigen

A

Its low affinity and high avidity make it less specific but highly effective at binding to invading antigens

IgM is an excellent fixer of complement. Its pentameric form precludes IgM from leaving systemic circulation. Initial production of IgM has lower affinity but higher avidity due to multiple binding sites. This provides less opportunities for the antigen to escape. Monomeric IgM binds to B cells, not mast cells.

How well did you know this?
1
Not at all
2
3
4
5
Perfectly
6
Q

Which of the following interleukins that are produced by macrophages stimulate T helper 1 cell activity?

1 IL-1
2 IL-6
3 IL-12
4 TNF-α
5 IL-4

A

IL-12

IL-1 stimulates T helper 2 cell activity. IL-6 stimulate antibody production by B cells. IL-12 stimulates Th1 cells to produce IL-2 and IFN-α. TNF-α is not an interleukin. IL-4 is produced by Th2 cells.

How well did you know this?
1
Not at all
2
3
4
5
Perfectly
7
Q

In which of the following scenarios will tolerance to a foreign antigen be induced in T cells?

1 Binding of antigen by the TCR, but lack of secondary signal mediated through CD 28
2 During development, binding of MHC expressed on thymocytes by TCR on developing T cells
3 Specific antigens such as pneumococcal polysaccharide block the TCR, preventing further activation
4 Repeated exposure to high concentrations of antigen will result in clonal exhaustion
5 Addition of an antigen with an adjuvant will induce tolerance

A

Binding of antigen by the TCR, but lack of secondary signal mediated through CD 28

Binding of TCR and CD 28 is required for activation of a T cell. If TCR only is bound, the T cell will not respond to the antigen. While recognition of MHC on thymocytes is essential for T cells to become tolerant, this tolerance is to self-antigens. Certain antigens such as pneumococcal antigen or repeated exposure to high concentrations of antigen will induce tolerance in B cells, and have not been demonstrated to induce tolerance in T cells. Addition of an adjuvant to an antigen is a way to break tolerance.

How well did you know this?
1
Not at all
2
3
4
5
Perfectly
8
Q

Which of the following is an example of Type II hypersensitivity?

1 Acute serum sickness
2 Chronic serum sickness
3 Hemolytic disease of the newborn
4 Hypersensitivity pneumonitis
5 Metal allergies

A

Hemolytic disease of the newborn

Type II hypersensitivity is usually mediated through antibodies and complement and is directed against erythrocytes. Acute & chronic serum sickness as well as hypersensitivity pneumonitis are initiated by immune complex deposition within the walls of a capillary bed, and are examples of type III hypersensitivity. Metal allergies are classified as type IV hypersensitivity.

How well did you know this?
1
Not at all
2
3
4
5
Perfectly
9
Q

A delayed hypersensitivity reaction is characterized by

1 An infiltrate composed of eosinophils
2 An infiltrate composed of lymphocytes and macrophages
3 An infiltrate composed of neutrophils
4 Edema without cellular infiltrate

A

An infiltrate composed of lymphocytes and macrophages

Delayed hypersensitivity is a function of helper (CD4) T lymphocytes. The response starts hours after contact with antigen. It consists of mainly mononuclear cell infiltration (macrophages and helper [CD4] T cells) and tissue induration, an example of which is the tuberculin test.

How well did you know this?
1
Not at all
2
3
4
5
Perfectly
10
Q

Acute poststreptococcal glomerulonephritis is characterized by:

1 Low complement levels and lumpy deposits of immunoglobulin and C3 seen along the glomerular basement membranes
2 High levels of complement
3 Intravascular clumping of platelets
4 Tissue induration

A

Low complement levels and lumpy deposits of immunoglobulin and C3 seen along the glomerular basement membranes

Acute poststreptococcal glomerulonephritis is a well-accepted immune-complex disease. Its onset follows several weeks after a group A beta-hemolytic streptococcal infection esp with nephritogenic serotypes of Streptococcus pyogenes . The complement levels are low and lumpy deposits of immunoglobulin and C3 are seen along glomerular basement membranes by immunofluorescence suggesting the presence of antigen-antibody complexes. It is postulated that streptococcal antigen-antibody complexes, after being deposited on glomeruli, fix complement and attract neutrophils, which start the inflammatory process.

How well did you know this?
1
Not at all
2
3
4
5
Perfectly
11
Q

The principal difference between cytotoxic (type II) and immune complex (type III) hypersensitivity is

1 The participation of T cells
2 The participation of complement
3 The distribution of antigen-antibody complexes
4 The class (isotype) of antibody

A

The distribution of antigen-antibody complexes

Cytotoxic hypersensitivity occurs when antibody directed at antigens of the cell membrane activates the complement. This generates a membrane attack complex, which damages the cell membrane. Immune-complex hypersensitivity occurs when antigen-antibody complexes induce an inflammatory response in tissues. The immune complexes occasionally persist and are deposited in tissues, resulting in several disorders.

How well did you know this?
1
Not at all
2
3
4
5
Perfectly
12
Q

AIDS is caused by a human retrovirus that kills

1 CD8-positive T lymphocytes
2 CD4-positive T lymphocytes
3 Lymphocyte stem cells
4 B lymphocytes

A

CD4-positive T lymphocytes

Patients with AIDS have greatly reduced helper T cell numbers caused by infection with the retrovirus that specifically infects cells with the CD4 receptor. The HIV virus has type-specific envelope glycoproteins called gp120 and gp41. gp120 protrudes from the surface and interacts with the CD4 receptor on the cell surface. gp41 is embedded in the envelope and mediates the fusion of the viral envelope with the cell membrane at the time of infection.

How well did you know this?
1
Not at all
2
3
4
5
Perfectly
13
Q

An 11-year-old boy goes on his first camping trip to a National forest with his scout troop. Upon arrival, he eagerly hikes through the forest to the campout area to pitch his tent. The next day he complains of intense itching and has blistering on his hands, arms and legs. What is the most likely diagnosis?

1 Serum sickness
2 Hay fever
3 Arthus reaction
4 Contact dermatitis
5 Farmer’s lung disease

A

Contact dermatitis

The boy likely has allergic contact dermatitis (contact hypersensitivity) where he may have encountered poison oak or poison ivy in the forest or in the camp area. Contact dermatitis is a cell-mediated, type IV hypersensitivity reaction. Poison ivy or oak produces urushiols (small molecular weight oils on the leaves that act as immunogens) that penetrate the skin and act as haptens after binding to large carrier proteins to become complete antigens. No one is born with sensitivity to poison ivy but if exposed enough most people become sensitized and remain allergic. The sensitized person develops erythema, itching fluid-filled vesicles, and sometimes necrosis generally seen within 12-48 hours after contact.
Hay fever, or allergic rhinitis, is usually caused by pollen spores, and can occur within minutes of exposure to an allergen.
Serum sickness is an example of type III hypersensitivity and is an immune complex disease in which antibodies to foreign proteins are produced. Serum sickness is now caused by drugs (not serum) and is manifested by fever, itching, arthralgias, proteinuria, and lymphadenopathy which occur 5-10 days after antigen exposure.
Arthus reaction is a local subacute antibody-mediated type III hypersensitivity reaction. Intradermal injection of the antigen induces antibodies which form antigen-antibody complexes in the skin causing edema, necrosis, and activation of complement. An example is hypersensitivity pneumonitis.
Farmer’s lung disease results from inhalation of spores from Actinomycetes species such as in moldy hay. Early lesions are thought to be an immune complex type III hypersensitivity followed by granulomatous interstitial inflammation of the lungs characteristic of type IV cell-mediated responses. Farmer’s lung disease is the most frequently reported type of hypersensitivity pneumonitis.

How well did you know this?
1
Not at all
2
3
4
5
Perfectly
14
Q

Which of the following statements about processing of exogenous and endogenous antigen is correct?

1 Exogenous antigen is processed by B cells, macrophages, and dendritic cells and presented to T cells on MHC class I molecules
2 Endogenous antigen is processed by B cells, macrophages, and dendritic cells and presented to T cells on MHC class I molecules
3 Endogenous antigen is processed by body cells and presented to B cells on MHC class II molecules
4 Endogenous antigen is processed by body cells and presented to T cells on MHC class I molecules
5 Exogenous antigen is processed by body cells and presented to T cells on MHC class II molecules

A

Endogenous antigen is processed by body cells and presented to T cells on MHC class I molecules

Exogenous antigen is antigen taken up by antigen-presenting cells (macrophages, B cells, and dendritic cells) and presented on MHC class II molecules to T cells. Endogenous antigen is processed by cells of the body and presented to CD 8 T cells on MHC class I molecules. Expression of MHC class II is limited to cells of the immune system.

How well did you know this?
1
Not at all
2
3
4
5
Perfectly
15
Q

Which of the following best describes thymic independent antigens?

1 Somatic antigens that are presented outside the thymus
2 Antigens that activate the immune response without relying upon T cells
3 Antigens that are presented to T cells in sites other than the thymus
4 Antigens that induce the immune response in cells that mature within the thymus
5 Antigens that are protein in nature

A

Antigens that activate the immune response without relying upon T cells

Thymic independent antigens are antigens that stimulate an immune response without the presence of T helper cells. These are often bacterial carbohydrates such as pneumococcal polysaccharides.

How well did you know this?
1
Not at all
2
3
4
5
Perfectly
16
Q

CD 4+ lymphocytes play an active role in all but which of the following functions?

1 Activate cytotoxic T cells
2 Activate macrophages
3 Stimulate B cells to produce antibody
4 Present antigen
5 Secrete lymphokines

A

Present antigen

Antigen presentation is carried out by B cells, macrophages, and dendritic cells. CD 4 + T cells are “helper” cells that serve to activate both humoral and cell-mediated immunity.

How well did you know this?
1
Not at all
2
3
4
5
Perfectly
17
Q

CD 8+ lymphocytes play an active role in all but which of the following functions?

1 Lyse virally infected cells
2 Lyse tumor cells
3 Stimulate B cells to produce antibody
4 Suppress inappropriate immune
responses
5 Secrete lymphokines

A

Stimulate B cells to produce antibody

CD 8 + lymphocytes use MHC class I molecules to check for the presence of endogenous antigens. These antigens are produced in virally infected cells as well as is some tumor cells. Additionally, CD 8 + cells play a major role in avoidance of autoimmunity by either direct suppression or lysis of immune cells that respond to autoantigens. These cells secrete lymphokines as part of the immune response

How well did you know this?
1
Not at all
2
3
4
5
Perfectly
18
Q

B cells play an active role in all but which of the following functions?

1 Production of antibody
2 Secretion of lymphokines
3 Presentation of antigen
4 Induction of Type I hypersensitivity
5 Induction of Type IV hypersensitivity

A

Induction of Type IV hypersensitivity

B cells are primarily producers of antibodies. They do present antigen and secret low levels of lymphokines when doing so. IgE produced by B cells is instrumental in Type I hypersensitivity, or anaphylaxis. Type IV hypersensitivity is not mediated by antibodies

How well did you know this?
1
Not at all
2
3
4
5
Perfectly
19
Q

The immunoglobulin IgA is found primarily in what locations in the body?

1 On mucosal surfaces
2 In the serum
3 On the surface of mast cells
4 In the aqueous humor
5 In CNS

A

On mucosal surfaces

IgA is found primarily on mucosal surfaces. Although there is small amounts in circulation, it is found bound to bilirubin and other substances.

How well did you know this?
1
Not at all
2
3
4
5
Perfectly
20
Q

Which lymphokine initiates switching from IgM/IgD production to IgE production in B cells?

1 IL-2
2 IL-4
3 IL-5
4 IFN-γ
5 TNF-γ

A

IL-4

Interleukin 4 initiates switching from production of IgM/IgD to IgE in B cells. This is blocked by IFN-γ. Immunotherapy, or “allergy shots,” attempts to stimulate Th1 cells to produce IFN-γ.

How well did you know this?
1
Not at all
2
3
4
5
Perfectly
21
Q

Identify the immunological disease that results in high levels of IgE, but also the inability of neutrophils to respond to chemotactic stimuli.

1 Job’s syndrome
2 Chediak-Higashi syndrome
3 Wiskott-Aldrich syndrome
4 Fanguys syndrome
5 X-linked granulomatous disease

A

Job’s syndrome

Defects in antigen processing cells and innate immune response often lead to a wide variety of syndromes. Job’s syndrome is characterized by neutrophils that are unable to respond to chemotaxins.

How well did you know this?
1
Not at all
2
3
4
5
Perfectly
22
Q

Identify the immunological disease that results from a deficiency in C1 esterase inhibitor resulting in the indirect generation of vasoactive kinins

1 X-linked agammaglobulinemia
2 Hereditary angioedema
3 Common variable hypogammaglobulinemia
4 Adenosine deaminase and nucleotide phosphorylase deficiency
5 DiGeorge syndrome

A

Hereditary angioedema

When C1 esterase inhibitor is not produced, excessive production of vasoactive kinins results. This causes angioedema.

How well did you know this?
1
Not at all
2
3
4
5
Perfectly
23
Q

Identify the immunological disease that results in the inability to produce a specific immunoglobulin, attributable to recurrent bacterial infections during adolescence

1 X-linked agammaglobulinemia
2 Hereditary angioedema
3 Common variable hypogammaglobulinemia
4 Adenosine deaminase and nucleotide phosphorylase deficiency
5 DiGeorge syndrome

A

Common variable hypogammaglobulinemia

It is thought that recurrent bacterial infections during young adulthood, in concert with exhaustion of plasma cells leads to the loss of the ability to produce effective, long-term antibody protection.

How well did you know this?
1
Not at all
2
3
4
5
Perfectly
24
Q

Which of the following best describes DiGeorge syndrome?

1 Mild facial hypoplasia with below average levels of circulating immunoglobulins
2 Mild facial hypoplasia with inability of macrophages to process antigen
3 Mild facial hypoplasia with deficiencies in T cell number and function
4 Mild facial hypoplasia with high levels of serumal IgE
5 Mild facial hypoplasia with low affinity, low avidity binding capacity of IgG

A

Mild facial hypoplasia with deficiencies in T cell number and function

DiGeorge syndrome is a congenital hypoplasia of the IIIrd and IV th pharyngeal arches resulting in facial, thymic and parathyroid hypoplasia. T cell numbers and functions are drastically decreased. Transplants of thymic tissues are currently being evaluated as a treatment modality for DiGeorge syndrome.

How well did you know this?
1
Not at all
2
3
4
5
Perfectly
25
Q

The following cytokine is involved in generating a major part of the Down syndrome phenotype.

1 Interferon
2 Interleukin
3 Plasminogen activator
4 Tumor necrosis factor

A

Interferon

The region responsible for most of the Down syndrome phenotype includes the genes for two subunits of the interferon α receptor. This leads to overactivity of interferon α action. The table below compares the Down Syndrome Anomalies with the interferon α side effects.
Down Syndrome
Anomalies Interferon Side Effects
Intellectual Disability Neurotoxicity, Memory Loss
Frontal lobe dysphasia Frontal lobe encephalopathy
Heart Anomalies Cardiotoxicity, Arrhythmia
Leukopenia Leukopenia
Autoimmune disease Autoimmune disease
Hypothyroidism Hypothyroidism
Hearing Loss Deafness
Short Stature Growth Inhibition

How well did you know this?
1
Not at all
2
3
4
5
Perfectly
26
Q

This cytokine is involved in the pathogenesis of Euthyroid Sick syndrome (low T3 syndrome) through its ability to inhibit type I 5’-deiodinase

1 Interferon
2 Interleukin
3 Plasminogen activator
4 Tumor necrosis factor

A

Tumor necrosis factor

In major acute or chronic illness, inflammatory cells will secrete cytokines such as tumor necrosis factors (TNF). TNF is capable of inhibiting type I 5’-deiodinase, allowing for acceleration of inner ring deiodination of thyroxine (T4). This leads to low circulating levels of triiodothyronine (T3), and increased levels of reverse triiodothyronine (rT3).

How well did you know this?
1
Not at all
2
3
4
5
Perfectly
27
Q

The smallest unit of a complex antigen capable of binding to an antibody is a(n)

1 Immunogen
2 Epitope
3 Hapten
4 Adjuvant
5 Carrier molecule

A

Epitope

Epitopes are an integral part of an antigen molecule that binds with an antibody. Epitopes are also known as antigenic determinants, as they determine the specificity of the antigen molecule. Epitopes can exist in sequential form or conformational form.
Haptens are small molecules that bind to carrier molecules and become immunogenic.
Adjuvants are substances that enhance the immunogenicity of molecules without changing their chemical structure.
A carrier molecule is an immunogenic molecule, such as a foreign protein, to which a hapten is coupled, thus, enabling the hapten to induce an immune response.

How well did you know this?
1
Not at all
2
3
4
5
Perfectly
28
Q

Antigens that are found in genetically different members of the same species are called

1 Heterologous antigens
2 Autologous antigens
3 Syngeneic antigens
4 Allogeneic antigens
5 Xenogeneic antigens

A

Allogeneic antigens

Allogeneic antigens are found in genetically different members of one’s own species
Heterologous antigen are also called xenogeneic antigens and are present in different species
Autologous antigens are self antigen found in an individual
Syngeneic antigens are found in genetically identical individuals

How well did you know this?
1
Not at all
2
3
4
5
Perfectly
29
Q

Which of the following antibodies is found in the highest concentration in serum?

1 IgA
2 IgD
3 IgE
4 IgG
5 IgM

A

IgG

IgG is the predominating antibody in serum and contributes to 75% to 85% to total immunoglobulin concentration. IgG is responsible for protection of the fetus, as it is the only immunoglobulin that can cross the placenta. It is divalent molecule and is made up of 2 H chains and 2 lights chain. IgG is divided into four subclasses: IgG1, IgG2, IgG3, and IgG4, according to antigenic differences in the H chain and the number and location of disulfide bonds. IgG is also involved in Rh disease of the newborn. It is produced in high concentration during secondary immune response.
IgM can fix the complement but cannot cross the placenta. It is the largest immunoglobulin and is a pentamer made up of 5 H2L2 units and 1 molecule of J chain. It has 10 antigen binding sites and has the highest avidity. It is first antibody to appear in an immune response.
IgA is produced by B cells and is found in high concentrations in tears, saliva, milk, and other secretions. It is a monomer made up of 1 4-chain unit.
IgE is a monomer that is present in very low concentration in serum. Increased level of IgE is found in antibody mediated allergic hypersensitivity and in parasitic infections. The mast cells and the basophils are involved in allergic response and have Fc receptors specific for IgE antibodies. The IgE antibodies serve as antigen receptor and can cause mast cells and basophils to release inflammatory mediators like histamine and heparin.
Ig D is a monomer comprised of 4-chain unit and is found on the surface of B lymphocytes. It is found in very low concentration in serum.
IgA, IgE, and IgD do not fix the complement and cannot cross the placenta.

How well did you know this?
1
Not at all
2
3
4
5
Perfectly
30
Q

The antibody that can cross the placenta and can fix the complement is

1 IgA
2 IgD
3 IgE
4 IgG
5 IgM

A

IgG

IgG is the predominating antibody in serum and contributes to 75% to 85% to total immunoglobulin concentration. IgG is responsible for protection of the fetus, as it is the only immunoglobulin that can cross the placenta. It is divalent molecule and is made up of 2 H chains and 2 lights chain. IgG is divided into four subclasses: IgG1, IgG2, IgG3, and IgG4, according to antigenic differences in the H chain and the number and location of disulfide bonds. IgG is also involved in Rh disease of the newborn. It is produced in high concentration during secondary immune response.
IgM can fix the complement but cannot cross the placenta. It is the largest immunoglobulin and is a pentamer made up of 5 H2L2 units and 1 molecule of J chain. It has 10 antigen binding sites and has the highest avidity. It is first antibody to appear in an immune response.
IgA is produced by B cells and is found in high concentrations in tears, saliva, milk, and other secretions. It is a monomer made up of 1 4-chain unit.
IgE is a monomer that is present in very low concentration in serum. Increased level of IgE is found in antibody mediated allergic hypersensitivity and in parasitic infections. The mast cells and the basophils are involved in allergic response and have Fc receptors specific for IgE antibodies. The IgE antibodies serve as antigen receptor and can cause mast cells and basophils to release inflammatory mediators like histamine and heparin.
Ig D is a monomer comprised of 4-chain unit and is found on the surface of B lymphocytes. It is found in very low concentration in serum.
IgA, IgE, and IgD do not fix the complement and cannot cross the placenta.

How well did you know this?
1
Not at all
2
3
4
5
Perfectly
31
Q

The antibody that is a pentamer and is first to appear in an immune response is:
Answer Choices

1 IgA
2 IgD
3 IgE
4 IgG
5 IgM

A

IgM

IgG is the predominating antibody in serum and contributes to 75% to 85% to total immunoglobulin concentration. IgG is responsible for protection of the fetus, as it is the only immunoglobulin that can cross the placenta. It is divalent molecule and is made up of 2 H chains and 2 lights chain. IgG is divided into four subclasses: IgG1, IgG2, IgG3, and IgG4, according to antigenic differences in the H chain and the number and location of disulfide bonds. IgG is also involved in Rh disease of the newborn. It is produced in high concentration during secondary immune response.
IgM can fix the complement but cannot cross the placenta. It is the largest immunoglobulin and is a pentamer made up of 5 H2L2 units and 1 molecule of J chain. It has 10 antigen binding sites and has the highest avidity. It is first antibody to appear in an immune response.
IgA is produced by B cells and is found in high concentrations in tears, saliva, milk, and other secretions. It is a monomer made up of 1 4-chain unit.
IgE is a monomer that is present in very low concentration in serum. Increased level of IgE is found in antibody mediated allergic hypersensitivity and in parasitic infections. The mast cells and the basophils are involved in allergic response and have Fc receptors specific for IgE antibodies. The IgE antibodies serve as antigen receptor and can cause mast cells and basophils to release inflammatory mediators like histamine and heparin.
Ig D is a monomer comprised of 4-chain unit and is found on the surface of B lymphocytes. It is found in very low concentration in serum.
IgA, IgE, and IgD do not fix the complement and cannot cross the placenta.

How well did you know this?
1
Not at all
2
3
4
5
Perfectly
32
Q

The antibody with 10 antigen binding sites is

1 IgA
2 IgD
3 IgE
4 IgG
5 IgM

A

IgM

IgG is the predominating antibody in serum and contributes to 75% to 85% to total immunoglobulin concentration. IgG is responsible for protection of the fetus, as it is the only immunoglobulin that can cross the placenta. It is divalent molecule and is made up of 2 H chains and 2 lights chain. IgG is divided into four subclasses: IgG1, IgG2, IgG3, and IgG4, according to antigenic differences in the H chain and the number and location of disulfide bonds. IgG is also involved in Rh disease of the newborn. It is produced in high concentration during secondary immune response.
IgM can fix the complement but cannot cross the placenta. It is the largest immunoglobulin and is a pentamer made up of 5 H2L2 units and 1 molecule of J chain. It has 10 antigen binding sites and has the highest avidity. It is first antibody to appear in an immune response.
IgA is produced by B cells and is found in high concentrations in tears, saliva, milk, and other secretions. It is a monomer made up of 1 4-chain unit.
IgE is a monomer that is present in very low concentration in serum. Increased level of IgE is found in antibody mediated allergic hypersensitivity and in parasitic infections. The mast cells and the basophils are involved in allergic response and have Fc receptors specific for IgE antibodies. The IgE antibodies serve as antigen receptor and can cause mast cells and basophils to release inflammatory mediators like histamine and heparin.
Ig D is a monomer comprised of 4-chain unit and is found on the surface of B lymphocytes. It is found in very low concentration in serum.
IgA, IgE, and IgD do not fix the complement and cannot cross the placenta.

How well did you know this?
1
Not at all
2
3
4
5
Perfectly
33
Q

The antibody predominant in milk, saliva, and tears is

1 IgA
2 IgD
3 IgE
4 IgG
5 IgM

A

IgA

IgG is the predominating antibody in serum and contributes to 75% to 85% to total immunoglobulin concentration. IgG is responsible for protection of the fetus, as it is the only immunoglobulin that can cross the placenta. It is divalent molecule and is made up of 2 H chains and 2 lights chain. IgG is divided into four subclasses: IgG1, IgG2, IgG3, and IgG4, according to antigenic differences in the H chain and the number and location of disulfide bonds. IgG is also involved in Rh disease of the newborn. It is produced in high concentration during secondary immune response.
IgM can fix the complement but cannot cross the placenta. It is the largest immunoglobulin and is a pentamer made up of 5 H2L2 units and 1 molecule of J chain. It has 10 antigen binding sites and has the highest avidity. It is first antibody to appear in an immune response.
IgA is produced by B cells and is found in high concentrations in tears, saliva, milk, and other secretions. It is a monomer made up of 1 4-chain unit.
IgE is a monomer that is present in very low concentration in serum. Increased level of IgE is found in antibody mediated allergic hypersensitivity and in parasitic infections. The mast cells and the basophils are involved in allergic response and have Fc receptors specific for IgE antibodies. The IgE antibodies serve as antigen receptor and can cause mast cells and basophils to release inflammatory mediators like histamine and heparin.
Ig D is a monomer comprised of 4-chain unit and is found on the surface of B lymphocytes. It is found in very low concentration in serum.
IgA, IgE, and IgD do not fix the complement and cannot cross the placenta.

How well did you know this?
1
Not at all
2
3
4
5
Perfectly
34
Q

What is the antibody responsible for mucous membrane protection from bacterial and virus attack?

1 IgA
2 IgD
3 IgE
4 IgG
5 IgM

A

IgA

IgG is the predominating antibody in serum and contributes to 75% to 85% to total immunoglobulin concentration. IgG is responsible for protection of the fetus, as it is the only immunoglobulin that can cross the placenta. It is divalent molecule and is made up of 2 H chains and 2 lights chain. IgG is divided into four subclasses: IgG1, IgG2, IgG3, and IgG4, according to antigenic differences in the H chain and the number and location of disulfide bonds. IgG is also involved in Rh disease of the newborn. It is produced in high concentration during secondary immune response.
IgM can fix the complement but cannot cross the placenta. It is the largest immunoglobulin and is a pentamer made up of 5 H2L2 units and 1 molecule of J chain. It has 10 antigen binding sites and has the highest avidity. It is first antibody to appear in an immune response.
IgA is produced by B cells and is found in high concentrations in tears, saliva, milk, and other secretions. It is a monomer made up of 1 4-chain unit.
IgE is a monomer that is present in very low concentration in serum. Increased level of IgE is found in antibody mediated allergic hypersensitivity and in parasitic infections. The mast cells and the basophils are involved in allergic response and have Fc receptors specific for IgE antibodies. The IgE antibodies serve as antigen receptor and can cause mast cells and basophils to release inflammatory mediators like histamine and heparin.
Ig D is a monomer comprised of 4-chain unit and is found on the surface of B lymphocytes. It is found in very low concentration in serum.
IgA, IgE, and IgD do not fix the complement and cannot cross the placenta.

How well did you know this?
1
Not at all
2
3
4
5
Perfectly
35
Q

The high level of this antibody is found in people with immediate allergic hypersensitivity and is called

1 IgA
2 IgD
3 IgE
4 IgG
5 IgM

A

IgE

IgG is the predominating antibody in serum and contributes to 75% to 85% to total immunoglobulin concentration. IgG is responsible for protection of the fetus, as it is the only immunoglobulin that can cross the placenta. It is divalent molecule and is made up of 2 H chains and 2 lights chain. IgG is divided into four subclasses: IgG1, IgG2, IgG3, and IgG4, according to antigenic differences in the H chain and the number and location of disulfide bonds. IgG is also involved in Rh disease of the newborn. It is produced in high concentration during secondary immune response.
IgM can fix the complement but cannot cross the placenta. It is the largest immunoglobulin and is a pentamer made up of 5 H2L2 units and 1 molecule of J chain. It has 10 antigen binding sites and has the highest avidity. It is first antibody to appear in an immune response.
IgA is produced by B cells and is found in high concentrations in tears, saliva, milk, and other secretions. It is a monomer made up of 1 4-chain unit.
IgE is a monomer that is present in very low concentration in serum. Increased level of IgE is found in antibody mediated allergic hypersensitivity and in parasitic infections. The mast cells and the basophils are involved in allergic response and have Fc receptors specific for IgE antibodies. The IgE antibodies serve as antigen receptor and can cause mast cells and basophils to release inflammatory mediators like histamine and heparin.
Ig D is a monomer comprised of 4-chain unit and is found on the surface of B lymphocytes. It is found in very low concentration in serum.
IgA, IgE, and IgD do not fix the complement and cannot cross the placenta.

How well did you know this?
1
Not at all
2
3
4
5
Perfectly
36
Q

The antibody which mediates the release of histamine and heparin is

1 IgA
2 IgD
3 IgE
4 IgG
5 IgM

A

IgE

IgG is the predominating antibody in serum and contributes to 75% to 85% to total immunoglobulin concentration. IgG is responsible for protection of the fetus, as it is the only immunoglobulin that can cross the placenta. It is divalent molecule and is made up of 2 H chains and 2 lights chain. IgG is divided into four subclasses: IgG1, IgG2, IgG3, and IgG4, according to antigenic differences in the H chain and the number and location of disulfide bonds. IgG is also involved in Rh disease of the newborn. It is produced in high concentration during secondary immune response.
IgM can fix the complement but cannot cross the placenta. It is the largest immunoglobulin and is a pentamer made up of 5 H2L2 units and 1 molecule of J chain. It has 10 antigen binding sites and has the highest avidity. It is first antibody to appear in an immune response.
IgA is produced by B cells and is found in high concentrations in tears, saliva, milk, and other secretions. It is a monomer made up of 1 4-chain unit.
IgE is a monomer that is present in very low concentration in serum. Increased level of IgE is found in antibody mediated allergic hypersensitivity and in parasitic infections. The mast cells and the basophils are involved in allergic response and have Fc receptors specific for IgE antibodies. The IgE antibodies serve as antigen receptor and can cause mast cells and basophils to release inflammatory mediators like histamine and heparin.
Ig D is a monomer comprised of 4-chain unit and is found on the surface of B lymphocytes. It is found in very low concentration in serum.
IgA, IgE, and IgD do not fix the complement and cannot cross the placenta.

How well did you know this?
1
Not at all
2
3
4
5
Perfectly
37
Q

What is IgM

1 A pentamer
2 Has five antigen binding site
3 Can cross the placenta
4 Is predominant in secretion
5 Does not participate in agglutination reaction

A

A pentamer

IgM is a pentamer made up of five H2L2 units and one molecule of J chain. It has ten antigen binding sites and is excellent agglutinating antibody. It is the first antibody to appear in an immune response IgM can fix the complement and can kill the bacterial cell by cytolysis. IgM is the largest immunoglobulin and can not cross the placenta.
The predominating antibody in the secretions is IgA.

How well did you know this?
1
Not at all
2
3
4
5
Perfectly
38
Q

Which of the following forms an antigen binding site in an immunoglobulin molecule?

1 Amino terminal of each light chain of constant region
2 Carboxy terminal of constant heavy chain
3 Amino terminal of variable light and heavy chain
4 Carboxy terminal of variable heavy chain
5 Amino terminal of constant heavy and constant light chain

A

Amino terminal of variable light and heavy chain

Variable region of light chain and heavy chains consists of N-terminal that shows variation in amino acid sequences. Amino terminal of this region forms an antigen binding site.
Constant region consists of carboxy terminal and does not demonstrate variation in amino acid sequences. This region is responsible for biological functions.

How well did you know this?
1
Not at all
2
3
4
5
Perfectly
39
Q

Ataxia telangiectasia is

1 X-linked recessive syndrome
2 A combined defect of both cellular and humoral immunity
3 Characterized by elevated IgA levels
4 Associated with defective neutrophil function
5 An acquired C3 deficiency

A

A combined defect of both cellular and humoral immunity

Ataxia telangiectasia is an autosomal recessive syndrome characterized by a combined defect of both cellular and humoral immunities. Cerebellar ataxia (staggering) and telangiectasia (enlargement of blood vessels) especially affecting earlobe and conjunctiva are seen in this syndrome. Recurrent infections occur as antibody response to antigen is inhibited. Patient suffering from ataxia telangiectasia often demonstrates reduced levels of IgA, IgE, and IgG2, and decrease in circulating T lymphocytes.

How well did you know this?
1
Not at all
2
3
4
5
Perfectly
40
Q

Which of the following cells can retain the memory of prior contact with an antigen?

1 B lymphocytes
2 Macrophages
3 Basophils
4 Thrombocytes
5 Neutrophils

A

B lymphocytes

Lymphocytes are the only immunologically specific cellular component of the immune system. Lymphocytes are responsible for primary recognition of an antigen. Antigen binding to B lymphocyte results in the enlargement, proliferation, and differentiation of lymphocytes. Some of the proliferating cells do not differentiate into plasma cells but instead revert to a resting state to become memory B lymphocytes. These memory cells survive for years within lymphoid follicles and produce more memory and effector cells upon activation by antigens.
Macrophages are involved in phagocytosis, antigen presentation, and induction of immune response
Basophils consists of heparin and histamine in their granules and participate in acute and systemic hypersensitivity reactions.
Thrombocytes are also known as platelet and participate in the clotting mechanism of the body.
Neutrophils are the principle leukocytes associated with phagocytosis and localized inflammatory response.

How well did you know this?
1
Not at all
2
3
4
5
Perfectly
41
Q

Individuals of blood group type O

1 Have O antigen on their red blood cells
2 Have A antigen and B antigen on their red blood cells
3 Have neither A nor B antigen on their red blood cells
4 Lack anti-A and anti-B antibodies
5 Lack anti-A antibody

A

Have neither A nor B antigen on their red blood cells

Blood group antigens are found on red blood cells in the blood. Antibodies to specific blood group are found in all individual that lack that antigen. Type O individuals lack A and B antigens therefore they have both anti-A and anti-B antibodies. Hemagglutination reaction is used to type blood groups and match compatibility between donors and recipients for blood transfusion.
Type AB individuals have A antigen and B antigen on their red blood cells and therefore lack anti-A and anti-B antibodies.
Type A individuals have A antigens on their red blood cells and therefore lack anti-A antibody and contain anti-B antibody.
Type B individuals have B antigens on their red blood cells and therefore lack anti-B antibody and contain anti-A antibody.

How well did you know this?
1
Not at all
2
3
4
5
Perfectly
42
Q

Which of the following immunologic reaction involves combination of soluble antigen with soluble antibody to produce insoluble complexes?

1 Complement fixation
2 Agglutination
3 Immunofluorescence
4 Precipitation
5 Flocculation

A

Precipitation

Precipitation involves the interaction of soluble antigen with a soluble antibody in correct proportion, leading to a visible precipitate. Precipitation reaction is effected by the amount of antigen and antibody present. The largest amount of precipitate is found in zone of equivalence.
Complement fixation is triggering of the classical complement pathway resulting from the combination of antigen with specific antibody.
Agglutination reaction results when particulate antigens aggregate to form larger complexes in presence of specific antibody.
Flocculation is an antigen-antibody reaction where antigen is an insoluble particulate.
Immunofluorescence involves the use of fluorescence microscope and is used for antigen detection in cells or tissues through the use of specific antibody labeled with fluorescent compound.

How well did you know this?
1
Not at all
2
3
4
5
Perfectly
43
Q

Which of the following tests is used for confirmation of HIV infection?

1 Western blot assay
2 Enzyme linked immunosorbent assay
3 PPD test
4 RPR test
5 Rose-Waaler test

A

Western blot assay

Enzyme linked immunosorbent assay (ELISA) is used as a screening test for HIV infection. When results for ELISA for antibody to HIV is positive, western blot is used to determine if the results are true positive or false positive.
Western blot assay is used as a confirmatory test to detect antibodies to HIV-1, human immunodeficiency virus. Western blot is an adaptation of immunofixation electrophoresis and is used to detect the antibodies to organisms of complex antigenic composition. HIV antigens are electrophoresed in a gel, resulting in the separation of individual viral proteins. These viral proteins are then transferred to nitrocellulose paper by means of blotting, and the patient’s serum is applied over it. If antibodies are present, they bind to viral proteins and can be detected by addition of antibody to human IgG labeled with either radioactive material or an enzyme, which produces a visible color change.
Rose-Waller test is used for detection of rheumatoid factor.
PPD test or purified protein derivative test is a skin test available to detect sensitization with Mycobacterium tuberculosis
RPR test or rapid plasma reagin test is a screening test used for diagnosis of syphilis.

How well did you know this?
1
Not at all
2
3
4
5
Perfectly
44
Q

Which of the following methods is most sensitive for measurements of antigen and antibody?

1 Precipitation
2 Indirect enzyme linked immunosorbent assay
3 Bacterial agglutination
4 Immunoelectrophoresis
5 Passive hemagglutination

A

Indirect enzyme linked immunosorbent assay

Indirect or noncompetitive enzyme linked immunosorbent assay (ELISA) has higher sensitivity (0.0005μg/mL) and is used for qualitative and quantitative determination of unknown antibodies in patient’s specimen. In this method, an antigen is bound to solid phase, and unlabeled patient antibody is added and given time to react. Following the wash step, an enzyme labeled antiglobulin is added. The second labeled antibody reacts with the Fc portion of the patient’s antibody that is bound to solid phase. In absence of a specific antibody, there is no binding of second labeled antibody. In the wash step, all unbound labeled antibody is removed. In the next step, substrate is added and color development is measured. Color development is directly proportional to the amount of antibody in patient’s serum.
Indirect ELISA is more sensitive, and sensitivity of nanogram is achieved because all patient ligand has chance to participate in the reaction.
Sensitivity for precipitation and immunoelectrophoresis method is ~20μg/mL for antibody detection
Sensitivity for bacterial agglutination and passive hemagglutination methods is ~0.01μg/mL for antibody detection.

How well did you know this?
1
Not at all
2
3
4
5
Perfectly
45
Q

Three basic reaction patterns result from the relationship of antigen and antibody in Ouchterlony double diffusion technique. Which of the following observations is made with partial identity?

1 Lines of precipitation run parallel to each other
2 Lines of precipitation merge with spur formation
3 Lines of precipitation coming together forming a smooth curve
4 Lines of precipitation cross each other
5 Lines of precipitation form two concentric circles

A

Lines of precipitation merge with spur formation

In the Ouchterlony double diffusion technique, both antigen and antibody diffuse independently through the medium in vertical and horizontal dimensions. In this method, wells are cut into an agar plate, and antigens and antisera are placed in the wells. The diffusion results in formation of a visible precipitation band at the zone of equivalence. When an antibody is placed in the center well and antigens are placed in the surrounding wells, three basic reaction are possible depending on the relationship between the antigens and antibody.
Reaction of partial identity is indicated when precipitation line fuses with spur formation. This reaction occurs when antigens being tested are nonidentical but do share common determinants.
Reaction of identity is revealed when the precipitation lines meet at a point and form a smooth curve or arc. This type of precipitin band indicates that antibody is precipitating identical antigen specificities in each preparation. Lines of identity does not necessarily mean that the antigens are identical. It only represents serological identity or presence of common determinants.
Reaction of nonidentity is demonstrated by formation of precipitation lines crossing one another. Precipitin bands intersect each other, as they do not have common antigenic determinants.
Ouchterlony double diffusion technique is used for semi quantitative determinations of antigens or antibodies and to determine the purity and relatedness of the reactants.

How well did you know this?
1
Not at all
2
3
4
5
Perfectly
46
Q

Three basic reaction patterns result from the relationship of antigen and antibody in Ouchterlony double diffusion technique. Which of the following observations is made with identity?

1 Lines of precipitation run parallel to each other
2 Lines of precipitation merge with spur formation
3 Lines of precipitation coming together forming a smooth curve
4 Lines of precipitation cross each other
5 Lines of precipitation form two concentric circles

A

Lines of precipitation coming together forming a smooth curve

In the Ouchterlony double diffusion technique, both antigen and antibody diffuse independently through the medium in vertical and horizontal dimensions. In this method, wells are cut into an agar plate, and antigens and antisera are placed in the wells. The diffusion results in formation of a visible precipitation band at the zone of equivalence. When an antibody is placed in the center well and antigens are placed in the surrounding wells, three basic reaction are possible depending on the relationship between the antigens and antibody.
Reaction of partial identity is indicated when precipitation line fuses with spur formation. This reaction occurs when antigens being tested are nonidentical but do share common determinants.
Reaction of identity is revealed when the precipitation lines meet at a point and form a smooth curve or arc. This type of precipitin band indicates that antibody is precipitating identical antigen specificities in each preparation. Lines of identity does not necessarily mean that the antigens are identical. It only represents serological identity or presence of common determinants.
Reaction of nonidentity is demonstrated by formation of precipitation lines crossing one another. Precipitin bands intersect each other, as they do not have common antigenic determinants.
Ouchterlony double diffusion technique is used for semi quantitative determinations of antigens or antibodies and to determine the purity and relatedness of the reactants.

How well did you know this?
1
Not at all
2
3
4
5
Perfectly
47
Q

Three basic reaction patterns result from the relationship of antigen and antibody in Ouchterlony double diffusion technique. Which of the following observations is made with nonidentity?

1 Lines of precipitation run parallel to each other
2 Lines of precipitation merge with spur formation
3 Lines of precipitation coming together forming a smooth curve
4 Lines of precipitation cross each other
5 Lines of precipitation form two concentric circles

A

Lines of precipitation cross each other

In the Ouchterlony double diffusion technique, both antigen and antibody diffuse independently through the medium in vertical and horizontal dimensions. In this method, wells are cut into an agar plate, and antigens and antisera are placed in the wells. The diffusion results in formation of a visible precipitation band at the zone of equivalence. When an antibody is placed in the center well and antigens are placed in the surrounding wells, three basic reaction are possible depending on the relationship between the antigens and antibody.
Reaction of partial identity is indicated when precipitation line fuses with spur formation. This reaction occurs when antigens being tested are nonidentical but do share common determinants.
Reaction of identity is revealed when the precipitation lines meet at a point and form a smooth curve or arc. This type of precipitin band indicates that antibody is precipitating identical antigen specificities in each preparation. Lines of identity does not necessarily mean that the antigens are identical. It only represents serological identity or presence of common determinants.
Reaction of nonidentity is demonstrated by formation of precipitation lines crossing one another. Precipitin bands intersect each other, as they do not have common antigenic determinants.
Ouchterlony double diffusion technique is used for semi quantitative determinations of antigens or antibodies and to determine the purity and relatedness of the reactants.

How well did you know this?
1
Not at all
2
3
4
5
Perfectly
48
Q

Select the most correct statement concerning antigen antibody reaction.

1 Optimum precipitation occurs in area of antigen excess
2 Rocket immunoelectrophoresis is slower than radial immunodiffusion and can not be used for immunoglobulin quantitation
3 Immunoelectrophoresis procedure is used for identification of Bence Jones proteins in myeloma
4 Western blot test is an adaptation of passive agglutination technique
5 Nephelometry can not be used for quantitative measurements of immunoglobulins

A

Immunoelectrophoresis procedure is used for identification of Bence Jones proteins in myeloma

Immunoelectrophoresis procedure utilizes double diffusion and electrophoresis principles. Immunoelectrophoresis procedure is used for identification of free κ and λ chains in paraprotein disease. Using specific anti-κ and anti-λ antisera, monoclonal nature of Bence Jones proteins in myeloma can be determined.
The largest amount of precipitate is found in the zone of equivalence where the number of multivalent sites of antigen and antibody are approximately equal
Rocket immunoelectrophoresis applies electrophoresis for facilitating migration of antigen in the agar. It is much more rapid than radial immunodiffusion (RID) and results can be obtained in a few hours. RID is modification of single diffusion method and takes more than 18 hours to obtain the results. Rocket immunoelectrophoresis is used for quantitative measurements of immunoglobulins, alpha-feto-protein in amniotic fluid, and complement components in body fluid
Western blot test is an adaptation of immunofixation electrophoresis technique and is used as confirmatory test to detect antibodies to HIV-1
Nephelometry can be used for quantitative measurements of immunoglobulins, complement components, haptoglobin, C-reactive protein, and other serum proteins

How well did you know this?
1
Not at all
2
3
4
5
Perfectly
49
Q

Which of the following is true of prozone phenomenon?

1 It occurs in the zone of equivalence
2 It occurs in the zone of antibody excess
3 It occurs in zone of antigen excess
4 It results in false positive reaction
5 It results in maximum precipitation

A

It occurs in the zone of antibody excess

Formation of precipitation is highly dependent on relative concentration of antigen and antibody. Prozone phenomenon results from antibody excess and can lead to false negative reaction. In prozone phenomenon, antigen combines with only one or two antibody molecules failing to form a cross-linkage. When false negative reaction due to prozone phenomenon is suspected, the test may be repeated after dilution of antibody.
Excess of antigen leads to postzone phenomenon and gives false negative reaction.
Largest amount of precipitate is found in the zone of equivalence where number of multivalent sites of antigen and antibody are approximately equal.

How well did you know this?
1
Not at all
2
3
4
5
Perfectly
50
Q

Classical pathway is mediated by

1 Antigen antibody complex
2 Viral envelopes
3 Capsular polysaccharide
4 Endotoxins
5 Teichoic acids

A

Antigen antibody complex

Alternate pathway is activated by bacterial capsular polysaccharide, endotoxin, teichoic acids, and viral envelopes.
Classical pathway is mediated by antigen antibody complexes. The classes of immunoglobulin that can trigger classical pathway include IgM, IgG1, IgG2, and IgG3.
The complement proteins involved in classical pathway include C1q, C1r, C1s, C4, C2, C3, C5, C6, C7, C8, and C9. A single molecule of IgM or two cross-linked molecules of IgG are required to fix the complement. Antigen antibody complex binds to C1q subunit via receptor in Fc region of antibody and activates the complement cascade.

How well did you know this?
1
Not at all
2
3
4
5
Perfectly
51
Q

The complement component/complex associated with bacterial cell lysis is

1 C1q
2 C3b
3 C4b2b
4 C5a
5 C5b6789

A

C5b6789

C5b6789 forms a membrane attack complex. Insertion of C5b6789 complex into cell membrane causes disruption of osmotic balance, resulting in lysis of bacteria cells, red blood cells, and tumor cells.
C1 is composed of 3 subunits C1q, C1r, and C1s. C1q is an aggregate of polypeptides that binds to receptor on Fc region of IgG or IgM, and classical pathway of the complement system is activated. C4b2b is C3 convertase and cleaves the C3 molecule into C3a and C3b. C3b is directly involved with opsonization of bacteria. C5a is a potent anaphylotoxin and a chemotactic factor.

How well did you know this?
1
Not at all
2
3
4
5
Perfectly
52
Q

Hereditary angioedema is caused by deficiency of

1 C1q
2 C3
3 C1 inhibitor
4 Decay accelerating factor
5 Hagman factor

A

C1 inhibitor

C1 inhibitor, or C1INH, is a glycoprotein that binds to C1q, causing dissociation of C1r and C1s, which leads to inhibition of further enzymatic activity. C1INH also inhibit components required for clot formation, such as activated Hagman factor, plasmin, kallikrein, and activated factor XI.
Hereditary angioedema is caused by deficiency of C1INH. It is inherited as an autosomal dominant gene that codes for dysfunctional protein or an inactive protein. One normal gene cannot produce sufficient C1INH to keep up with the demand. Hereditary angioedema is characterized by recurrent attacks of angioedema, which effect the extremities, the skin, oropharynx, and other mucosal surfaces. Reduced levels of C4 and C2 and normal levels of C1q and C3 are found in hereditary angioedema. C1q and C3 levels are decreased in acquired C1INH deficiency.
Deficiency of C1INH leads to excess cleavage of C2 and C4 and perpetuation of classical pathway. This triggers increased production of kinin-related protein, causing increased vascular permeability. Continued swelling and edema occurs in response to localized antigen stimulus.

How well did you know this?
1
Not at all
2
3
4
5
Perfectly
53
Q

B-cell differentiation factor that enhances IgA switching best describes which of the following cytokines?

1 Interleukin-2
2 Interleukin-3
3 Interleukin-5
4 Interleukin-7
5 Interleukin-8
6 Interleukin-10
7 Interleukin-13

A

Interleukin-5

Interleukins are a group of cytokines that are polypeptide products of activated cells. Interleukins control a variety of cellular responses and contribute in regulation of immune response.
Lymphokines are primarily produced by activated T cells. Interleukin-2 (IL-2) is a T cell growth factor secreted by antigen-stimulated T helper cells. IL-2 causes proliferation and clonal expansion of activated T cells. IL-2 also enhances proliferation of B cells and IgG2 antibody production by B cells. IL-2 assists in the clearing of any foreign antigen from the area by enhancing cytotoxicity of natural killer cells (NK cells).
Interleukin-3 (IL-3) is a lymphokine secreted by activated T cells and is a major immunologic mediator of myeloid differentiation. IL-3 serves as an important link between the immune system and the hematopoietic system. IL-3 causes proliferation of granulocytes and macrophages. IL-3 is also associated with regulation and differentiation of megacaryocytic, erythrocytic, and mast cell precursors.
Interleukin-5 (IL-5), also known as B-cell differentiation factor, is produced by activated T helper cells and mast cells. IL-5 causes proliferation of activated B cells and increased eosinophil production. IL-5 also enhances IgM secretion and class switching of IgA.
Interleukin-7 (IL-7) is a hematopoietic growth factor secreted by bone marrow stromal cells. IL-7 functions as a growth factor for both T cell and B cell precursors. IL-7 also enhances cytotoxic activity of lymphocytes and monocytes.
Interleukin-8 (IL-8) is an inflammatory cytokine secreted by monocytes and macrophages. IL-8 is chemotactic for both neutrophils and T cells. IL-8 causes activation of neutrophils to release lysosomal enzymes and leads to superoxide dismutase production that is lethal for pathogens.
Interleukin-10 (IL-10) is produced by activated T cells and mast cells. IL-10 inhibits cytokine synthesis and T cell proliferation in the presence of monocytes. In particular, IL-10 inhibits interleukin -2 and interferon-γ production by T helper cells. IL-10 enhances B cell differentiation, leading to increased antibody response.
Interleukin-13 (IL-13) is produced by T helper cells. IL-13 is an important regulator of inflammatory response because it inhibits activity and release of inflammatory cytokines by macrophages.

How well did you know this?
1
Not at all
2
3
4
5
Perfectly
54
Q

Inhibitor of inflammatory cytokines secreted by macrophages best describes which of the following cytokines?

1 Interleukin-2
2 Interleukin-3
3 Interleukin-5
4 Interleukin-7
5 Interleukin-8
6 Interleukin-10
7 Interleukin-13

A

Interleukin-13

Interleukins are a group of cytokines that are polypeptide products of activated cells. Interleukins control a variety of cellular responses and contribute in regulation of immune response.
Lymphokines are primarily produced by activated T cells. Interleukin-2 (IL-2) is a T cell growth factor secreted by antigen-stimulated T helper cells. IL-2 causes proliferation and clonal expansion of activated T cells. IL-2 also enhances proliferation of B cells and IgG2 antibody production by B cells. IL-2 assists in the clearing of any foreign antigen from the area by enhancing cytotoxicity of natural killer cells (NK cells).
Interleukin-3 (IL-3) is a lymphokine secreted by activated T cells and is a major immunologic mediator of myeloid differentiation. IL-3 serves as an important link between the immune system and the hematopoietic system. IL-3 causes proliferation of granulocytes and macrophages. IL-3 is also associated with regulation and differentiation of megacaryocytic, erythrocytic, and mast cell precursors.
Interleukin-5 (IL-5), also known as B-cell differentiation factor, is produced by activated T helper cells and mast cells. IL-5 causes proliferation of activated B cells and increased eosinophil production. IL-5 also enhances IgM secretion and class switching of IgA.
Interleukin-7 (IL-7) is a hematopoietic growth factor secreted by bone marrow stromal cells. IL-7 functions as a growth factor for both T cell and B cell precursors. IL-7 also enhances cytotoxic activity of lymphocytes and monocytes.
Interleukin-8 (IL-8) is an inflammatory cytokine secreted by monocytes and macrophages. IL-8 is chemotactic for both neutrophils and T cells. IL-8 causes activation of neutrophils to release lysosomal enzymes and leads to superoxide dismutase production that is lethal for pathogens.
Interleukin-10 (IL-10) is produced by activated T cells and mast cells. IL-10 inhibits cytokine synthesis and T cell proliferation in the presence of monocytes. In particular, IL-10 inhibits interleukin -2 and interferon-γ production by T helper cells. IL-10 enhances B cell differentiation, leading to increased antibody response.
Interleukin-13 (IL-13) is produced by T helper cells. IL-13 is an important regulator of inflammatory response because it inhibits activity and release of inflammatory cytokines by macrophages.

How well did you know this?
1
Not at all
2
3
4
5
Perfectly
55
Q

Cytokines produced by activated T cells and responsible for differentiation of hematopoietic stem cells and mast cells best describes which of the following cytokines?

1 Interleukin-2
2 Interleukin-3
3 Interleukin-5
4 Interleukin-7
5 Interleukin-8
6 Interleukin-10
7 Interleukin-13

A

Interleukin-3

Interleukins are a group of cytokines that are polypeptide products of activated cells. Interleukins control a variety of cellular responses and contribute in regulation of immune response.
Lymphokines are primarily produced by activated T cells. Interleukin-2 (IL-2) is a T cell growth factor secreted by antigen-stimulated T helper cells. IL-2 causes proliferation and clonal expansion of activated T cells. IL-2 also enhances proliferation of B cells and IgG2 antibody production by B cells. IL-2 assists in the clearing of any foreign antigen from the area by enhancing cytotoxicity of natural killer cells (NK cells).
Interleukin-3 (IL-3) is a lymphokine secreted by activated T cells and is a major immunologic mediator of myeloid differentiation. IL-3 serves as an important link between the immune system and the hematopoietic system. IL-3 causes proliferation of granulocytes and macrophages. IL-3 is also associated with regulation and differentiation of megacaryocytic, erythrocytic, and mast cell precursors.
Interleukin-5 (IL-5), also known as B-cell differentiation factor, is produced by activated T helper cells and mast cells. IL-5 causes proliferation of activated B cells and increased eosinophil production. IL-5 also enhances IgM secretion and class switching of IgA.
Interleukin-7 (IL-7) is a hematopoietic growth factor secreted by bone marrow stromal cells. IL-7 functions as a growth factor for both T cell and B cell precursors. IL-7 also enhances cytotoxic activity of lymphocytes and monocytes.
Interleukin-8 (IL-8) is an inflammatory cytokine secreted by monocytes and macrophages. IL-8 is chemotactic for both neutrophils and T cells. IL-8 causes activation of neutrophils to release lysosomal enzymes and leads to superoxide dismutase production that is lethal for pathogens.
Interleukin-10 (IL-10) is produced by activated T cells and mast cells. IL-10 inhibits cytokine synthesis and T cell proliferation in the presence of monocytes. In particular, IL-10 inhibits interleukin -2 and interferon-γ production by T helper cells. IL-10 enhances B cell differentiation, leading to increased antibody response.
Interleukin-13 (IL-13) is produced by T helper cells. IL-13 is an important regulator of inflammatory response because it inhibits activity and release of inflammatory cytokines by macrophages.

How well did you know this?
1
Not at all
2
3
4
5
Perfectly
56
Q

Lymphokine produced by antigen-stimulated T helper cells, causes proliferation of activated T and B cells and enhances toxicity of NK cells best describes which of the following cytokines?

1 Interleukin-2
2 Interleukin-3
3 Interleukin-5
4 Interleukin-7
5 Interleukin-8
6 Interleukin-10
7 Interleukin-13

A

Interleukin-2

Interleukins are a group of cytokines that are polypeptide products of activated cells. Interleukins control a variety of cellular responses and contribute in regulation of immune response.
Lymphokines are primarily produced by activated T cells. Interleukin-2 (IL-2) is a T cell growth factor secreted by antigen-stimulated T helper cells. IL-2 causes proliferation and clonal expansion of activated T cells. IL-2 also enhances proliferation of B cells and IgG2 antibody production by B cells. IL-2 assists in the clearing of any foreign antigen from the area by enhancing cytotoxicity of natural killer cells (NK cells).
Interleukin-3 (IL-3) is a lymphokine secreted by activated T cells and is a major immunologic mediator of myeloid differentiation. IL-3 serves as an important link between the immune system and the hematopoietic system. IL-3 causes proliferation of granulocytes and macrophages. IL-3 is also associated with regulation and differentiation of megacaryocytic, erythrocytic, and mast cell precursors.
Interleukin-5 (IL-5), also known as B-cell differentiation factor, is produced by activated T helper cells and mast cells. IL-5 causes proliferation of activated B cells and increased eosinophil production. IL-5 also enhances IgM secretion and class switching of IgA.
Interleukin-7 (IL-7) is a hematopoietic growth factor secreted by bone marrow stromal cells. IL-7 functions as a growth factor for both T cell and B cell precursors. IL-7 also enhances cytotoxic activity of lymphocytes and monocytes.
Interleukin-8 (IL-8) is an inflammatory cytokine secreted by monocytes and macrophages. IL-8 is chemotactic for both neutrophils and T cells. IL-8 causes activation of neutrophils to release lysosomal enzymes and leads to superoxide dismutase production that is lethal for pathogens.
Interleukin-10 (IL-10) is produced by activated T cells and mast cells. IL-10 inhibits cytokine synthesis and T cell proliferation in the presence of monocytes. In particular, IL-10 inhibits interleukin -2 and interferon-γ production by T helper cells. IL-10 enhances B cell differentiation, leading to increased antibody response.
Interleukin-13 (IL-13) is produced by T helper cells. IL-13 is an important regulator of inflammatory response because it inhibits activity and release of inflammatory cytokines by macrophages.

How well did you know this?
1
Not at all
2
3
4
5
Perfectly
57
Q

Neutrophil and T cell chemoattractant produced by monocytes and macrophages best describes which of the following cytokines?

1 Interleukin-2
2 Interleukin-3
3 Interleukin-5
4 Interleukin-7
5 Interleukin-8
6 Interleukin-10
7 Interleukin-13

A

Interleukin-8

Interleukins are a group of cytokines that are polypeptide products of activated cells. Interleukins control a variety of cellular responses and contribute in regulation of immune response.
Lymphokines are primarily produced by activated T cells. Interleukin-2 (IL-2) is a T cell growth factor secreted by antigen-stimulated T helper cells. IL-2 causes proliferation and clonal expansion of activated T cells. IL-2 also enhances proliferation of B cells and IgG2 antibody production by B cells. IL-2 assists in the clearing of any foreign antigen from the area by enhancing cytotoxicity of natural killer cells (NK cells).
Interleukin-3 (IL-3) is a lymphokine secreted by activated T cells and is a major immunologic mediator of myeloid differentiation. IL-3 serves as an important link between the immune system and the hematopoietic system. IL-3 causes proliferation of granulocytes and macrophages. IL-3 is also associated with regulation and differentiation of megacaryocytic, erythrocytic, and mast cell precursors.
Interleukin-5 (IL-5), also known as B-cell differentiation factor, is produced by activated T helper cells and mast cells. IL-5 causes proliferation of activated B cells and increased eosinophil production. IL-5 also enhances IgM secretion and class switching of IgA.
Interleukin-7 (IL-7) is a hematopoietic growth factor secreted by bone marrow stromal cells. IL-7 functions as a growth factor for both T cell and B cell precursors. IL-7 also enhances cytotoxic activity of lymphocytes and monocytes.
Interleukin-8 (IL-8) is an inflammatory cytokine secreted by monocytes and macrophages. IL-8 is chemotactic for both neutrophils and T cells. IL-8 causes activation of neutrophils to release lysosomal enzymes and leads to superoxide dismutase production that is lethal for pathogens.
Interleukin-10 (IL-10) is produced by activated T cells and mast cells. IL-10 inhibits cytokine synthesis and T cell proliferation in the presence of monocytes. In particular, IL-10 inhibits interleukin -2 and interferon-γ production by T helper cells. IL-10 enhances B cell differentiation, leading to increased antibody response.
Interleukin-13 (IL-13) is produced by T helper cells. IL-13 is an important regulator of inflammatory response because it inhibits activity and release of inflammatory cytokines by macrophages.

How well did you know this?
1
Not at all
2
3
4
5
Perfectly
58
Q

Hematopoietic growth factor secreted by bone marrow stromal cells, associated with proliferation of pre-B cells, pre-T cells, and mature T cells best describes which of the following cytokines?

1 Interleukin-2
2 Interleukin-3
3 Interleukin-5
4 Interleukin-7
5 Interleukin-8
6 Interleukin-10
7 Interleukin-13

A

Interleukin-7

Interleukins are a group of cytokines that are polypeptide products of activated cells. Interleukins control a variety of cellular responses and contribute in regulation of immune response.
Lymphokines are primarily produced by activated T cells. Interleukin-2 (IL-2) is a T cell growth factor secreted by antigen-stimulated T helper cells. IL-2 causes proliferation and clonal expansion of activated T cells. IL-2 also enhances proliferation of B cells and IgG2 antibody production by B cells. IL-2 assists in the clearing of any foreign antigen from the area by enhancing cytotoxicity of natural killer cells (NK cells).
Interleukin-3 (IL-3) is a lymphokine secreted by activated T cells and is a major immunologic mediator of myeloid differentiation. IL-3 serves as an important link between the immune system and the hematopoietic system. IL-3 causes proliferation of granulocytes and macrophages. IL-3 is also associated with regulation and differentiation of megacaryocytic, erythrocytic, and mast cell precursors.
Interleukin-5 (IL-5), also known as B-cell differentiation factor, is produced by activated T helper cells and mast cells. IL-5 causes proliferation of activated B cells and increased eosinophil production. IL-5 also enhances IgM secretion and class switching of IgA.
Interleukin-7 (IL-7) is a hematopoietic growth factor secreted by bone marrow stromal cells. IL-7 functions as a growth factor for both T cell and B cell precursors. IL-7 also enhances cytotoxic activity of lymphocytes and monocytes.
Interleukin-8 (IL-8) is an inflammatory cytokine secreted by monocytes and macrophages. IL-8 is chemotactic for both neutrophils and T cells. IL-8 causes activation of neutrophils to release lysosomal enzymes and leads to superoxide dismutase production that is lethal for pathogens.
Interleukin-10 (IL-10) is produced by activated T cells and mast cells. IL-10 inhibits cytokine synthesis and T cell proliferation in the presence of monocytes. In particular, IL-10 inhibits interleukin -2 and interferon-γ production by T helper cells. IL-10 enhances B cell differentiation, leading to increased antibody response.
Interleukin-13 (IL-13) is produced by T helper cells. IL-13 is an important regulator of inflammatory response because it inhibits activity and release of inflammatory cytokines by macrophages.

How well did you know this?
1
Not at all
2
3
4
5
Perfectly
59
Q

Which of the following is true of cytokines?

1 Tumor necrosis factors stimulate T cell proliferation
2 Tumor necrosis factor-α is secreted by macrophages
3 Tumor necrosis factor-β is secreted by bone marrow stromal cells
4 Interferon-β has antiviral activity and inhibits MHC class I expression
5 Transforming growth factor-β enhances B cell proliferation and suppresses secretion of inflammatory mediators

A

Tumor necrosis factor-α is secreted by macrophages

Tumor necrosis factors (TNF) possess many of the same activity as IL-1. TNF differs from IL-1 in that TNF are not able to stimulate T cell proliferation, and IL-1 has only limited ability to lyse tumor cells.
Tumor necrosis factor-α (TNF-α) is secreted by macrophages when stimulated by antigen. TNF-α is associated with production of hemorrhagic necrosis in tumors.
Tumor necrosis factor-β (TNF-β) is produced by T cells and B cells. TNF- β is responsible for killing tumor cells and virally infected cells.
Interferon-β inhibits synthesis of viral proteins by triggering the host cell to produce a second protein that interferes with the viral protein synthesis. Interferon-β is primarily secreted by fibroblasts and epithelial cells and enhances MHC class I expression.
Transforming growth factor-β (TGF-β) inhibits T and B cell proliferation. TGF-β is chemotactic for monocytes and induces secretion of inflammatory mediators.

How well did you know this?
1
Not at all
2
3
4
5
Perfectly
60
Q

T cells

1 Are thymus independent lymphocytes
2 Are responsible for cellular immune response
3 Lack antigen receptors on the cell surface
4 Differentiates into plasma cells
5 Consist of IgM on their surface

A

Are responsible for cellular immune response

T cells are thymus dependent lymphocytes as they mature in thymus. B cells are thymus independent lymphocytes that develops in the bursa of Fabricius. IgM is present on the surface of B cells and not T cell. Also B cells are associated with antibody production. B cells differentiates into plasma cells and memory cells. Specific antigen receptors are present on the cell surface of T cells and are involved in cell mediated immunity. T cells are associated with production of cytokines which include synthesis of IL-2, IL-4, IL-5 and gamma interferons.

How well did you know this?
1
Not at all
2
3
4
5
Perfectly
61
Q

Which of the following surface markers can be described as a surface antigen found on T helper cells?

1 CD2
2 CD3
3 CD4
4 CD8
5 CD19

A

CD4

Proteins found on cell surfaces can be used as markers to differentiate T cells and B cells. The maturation stages of the two types of cells can also be distinguished using these antigens or surface markers. CD (cluster of differentiation) classification is used as a reference in standardizing names of membrane proteins found on all human white cells.
CD2 is the earliest T-cell marker and is present on every peripheral T cells. It is also expressed by NK cells. CD2 is a receptor for sheep red blood cells. It demonstrates the rosetting phenomenon, in which sheep red blood cells adhere to CD2 receptor, encircling the T cells and forming a daisy pattern.
Mature T cells bearing the CD4 receptor are known as helper or inducer cells. CD4+ cells recognize antigens in association with MHC class II product.
The population of matured T cells that express CD8 antigens are T-cytotoxic cells. These cells provide primary line of defense against changed self antigen, especially virally infected cells and tumor cells. Direct cell contact can result in destruction of target cells without antibody involvement.
CD3 serves as T cell antigen receptor and is involved in signal transduction.
CD19 is found on B cells and acts as a co-receptor that assists in enhancing antibody production in mature B cells.

How well did you know this?
1
Not at all
2
3
4
5
Perfectly
62
Q

Which of the following surface markers can be described as a T cell receptor responsible for sheep red blood cell rosetting?

1 CD2
2 CD3
3 CD4
4 CD8
5 CD19

A

CD2

Proteins found on cell surfaces can be used as markers to differentiate T cells and B cells. The maturation stages of the two types of cells can also be distinguished using these antigens or surface markers. CD (cluster of differentiation) classification is used as a reference in standardizing names of membrane proteins found on all human white cells.
CD2 is the earliest T-cell marker and is present on every peripheral T cells. It is also expressed by NK cells. CD2 is a receptor for sheep red blood cells. It demonstrates the rosetting phenomenon, in which sheep red blood cells adhere to CD2 receptor, encircling the T cells and forming a daisy pattern.
Mature T cells bearing the CD4 receptor are known as helper or inducer cells. CD4+ cells recognize antigens in association with MHC class II product.
The population of matured T cells that express CD8 antigens are T-cytotoxic cells. These cells provide primary line of defense against changed self antigen, especially virally infected cells and tumor cells. Direct cell contact can result in destruction of target cells without antibody involvement.
CD3 serves as T cell antigen receptor and is involved in signal transduction.
CD19 is found on B cells and acts as a co-receptor that assists in enhancing antibody production in mature B cells.

How well did you know this?
1
Not at all
2
3
4
5
Perfectly
63
Q

Which of the following is a surface marker present primarily on T-cytotoxic cells and T-suppressor cells

1 CD2
2 CD3
3 CD4
4 CD8
5 CD19

A

CD8

Proteins found on cell surfaces can be used as markers to differentiate T cells and B cells. The maturation stages of the two types of cells can also be distinguished using these antigens or surface markers. CD (cluster of differentiation) classification is used as a reference in standardizing names of membrane proteins found on all human white cells.
CD2 is the earliest T-cell marker and is present on every peripheral T cells. It is also expressed by NK cells. CD2 is a receptor for sheep red blood cells. It demonstrates the rosetting phenomenon, in which sheep red blood cells adhere to CD2 receptor, encircling the T cells and forming a daisy pattern.
Mature T cells bearing the CD4 receptor are known as helper or inducer cells. CD4+ cells recognize antigens in association with MHC class II product.
The population of matured T cells that express CD8 antigens are T-cytotoxic cells. These cells provide primary line of defense against changed self antigen, especially virally infected cells and tumor cells. Direct cell contact can result in destruction of target cells without antibody involvement.
CD3 serves as T cell antigen receptor and is involved in signal transduction.
CD19 is found on B cells and acts as a co-receptor that assists in enhancing antibody production in mature B cells.

How well did you know this?
1
Not at all
2
3
4
5
Perfectly
64
Q

Which of the following is a surface marker associated with T cell antigen receptor and signal transduction?

1 CD2
2 CD3
3 CD4
4 CD8
5 CD19

A

CD3

Proteins found on cell surfaces can be used as markers to differentiate T cells and B cells. The maturation stages of the two types of cells can also be distinguished using these antigens or surface markers. CD (cluster of differentiation) classification is used as a reference in standardizing names of membrane proteins found on all human white cells.
CD2 is the earliest T-cell marker and is present on every peripheral T cells. It is also expressed by NK cells. CD2 is a receptor for sheep red blood cells. It demonstrates the rosetting phenomenon, in which sheep red blood cells adhere to CD2 receptor, encircling the T cells and forming a daisy pattern.
Mature T cells bearing the CD4 receptor are known as helper or inducer cells. CD4+ cells recognize antigens in association with MHC class II product.
The population of matured T cells that express CD8 antigens are T-cytotoxic cells. These cells provide primary line of defense against changed self antigen, especially virally infected cells and tumor cells. Direct cell contact can result in destruction of target cells without antibody involvement.
CD3 serves as T cell antigen receptor and is involved in signal transduction.
CD19 is found on B cells and acts as a co-receptor that assists in enhancing antibody production in mature B cells.

How well did you know this?
1
Not at all
2
3
4
5
Perfectly
65
Q

Which of the following is a surface marker that acts as B cell co-receptor

1 CD2
2 CD3
3 CD4
4 CD8
5 CD19

A

CD19

Proteins found on cell surfaces can be used as markers to differentiate T cells and B cells. The maturation stages of the two types of cells can also be distinguished using these antigens or surface markers. CD (cluster of differentiation) classification is used as a reference in standardizing names of membrane proteins found on all human white cells.
CD2 is the earliest T-cell marker and is present on every peripheral T cells. It is also expressed by NK cells. CD2 is a receptor for sheep red blood cells. It demonstrates the rosetting phenomenon, in which sheep red blood cells adhere to CD2 receptor, encircling the T cells and forming a daisy pattern.
Mature T cells bearing the CD4 receptor are known as helper or inducer cells. CD4+ cells recognize antigens in association with MHC class II product.
The population of matured T cells that express CD8 antigens are T-cytotoxic cells. These cells provide primary line of defense against changed self antigen, especially virally infected cells and tumor cells. Direct cell contact can result in destruction of target cells without antibody involvement.
CD3 serves as T cell antigen receptor and is involved in signal transduction.
CD19 is found on B cells and acts as a co-receptor that assists in enhancing antibody production in mature B cells.

66
Q

Which of the following cell types eliminates virally infected cells and recognizes the antigen in conjunction with MHC class I molecule?

1 Kupffer cells
2 Eosinophils
3 Plasma cells
4 Cytotoxic T cells
5 Helper T cells
6 Natural killer cells

A

Cytotoxic T cells

Cytotoxic T cells are CD8+ with MHC class I restriction. Direct cell contact can result in destruction of target cells without antibody involvement. Cytotoxic T cells provide primary line of defense against changed self antigen, especially virally infected cells and tumor cells. Cytotoxic T cells recognize antigen in association with MHC class I molecule, which is expressed by all nucleated host cells.
Helper T cells are CD4+ cells and recognize the antigen in conjunction with MHC class II molecule. Helper T cells are required for antibody response to most antigens. These cells control proliferation, differentiation, and isotype switching of B cells.
Natural killer cells (NK cells) are large granular lymphocytes and do not express most of the recognizable surface marker of T or B cells. NK cells can mediate cytolytic reactions and kill target cells without prior exposure to antigen. Cytolytic activity of NK cells is nonphagocytic, extracellular, and not restricted by major histocompatibility complex (MHC). NK cells are also involved in antibody dependent cell mediated cytotoxicity (ADCC).
When B cells are stimulated by an antigen, they undergo differentiation and clonal expansion. Some of the transformed B cells eventually become antibody secreting plasma cells. Plasma cells are fully differentiated lymphocytes with abundant cytoplasmic immunoglobulins. Antibody production is the primary function of plasma cells.
Kupffer cells are fixed macrophages found in liver. These cells are associated with phagocytosis and antigen presentation.
Eosinophils are homeostatic regulators of inflammation and also play a key role in the host defense against some parasites, like nematodes. Eosinophils secrete cytotoxic enzymes in their granules when attached to the surface of the parasite via IgE and damage them by way of oxidative mechanism.

67
Q

Which of the following cell types synthesize and secrete immunoglobulins upon antigen stimulation?

1 Kupffer cells
2 Eosinophils
3 Plasma cells
4 Cytotoxic T cells
5 Helper T cells
6 Natural killer cells

A

Plasma cells

Cytotoxic T cells are CD8+ with MHC class I restriction. Direct cell contact can result in destruction of target cells without antibody involvement. Cytotoxic T cells provide primary line of defense against changed self antigen, especially virally infected cells and tumor cells. Cytotoxic T cells recognize antigen in association with MHC class I molecule, which is expressed by all nucleated host cells.
Helper T cells are CD4+ cells and recognize the antigen in conjunction with MHC class II molecule. Helper T cells are required for antibody response to most antigens. These cells control proliferation, differentiation, and isotype switching of B cells.
Natural killer cells (NK cells) are large granular lymphocytes and do not express most of the recognizable surface marker of T or B cells. NK cells can mediate cytolytic reactions and kill target cells without prior exposure to antigen. Cytolytic activity of NK cells is nonphagocytic, extracellular, and not restricted by major histocompatibility complex (MHC). NK cells are also involved in antibody dependent cell mediated cytotoxicity (ADCC).
When B cells are stimulated by an antigen, they undergo differentiation and clonal expansion. Some of the transformed B cells eventually become antibody secreting plasma cells. Plasma cells are fully differentiated lymphocytes with abundant cytoplasmic immunoglobulins. Antibody production is the primary function of plasma cells.
Kupffer cells are fixed macrophages found in liver. These cells are associated with phagocytosis and antigen presentation.
Eosinophils are homeostatic regulators of inflammation and also play a key role in the host defense against some parasites, like nematodes. Eosinophils secrete cytotoxic enzymes in their granules when attached to the surface of the parasite via IgE and damage them by way of oxidative mechanism.

68
Q

Which of the following cell types destroy target cells through an extracellular, MHC unrestricted, non phagocytic mechanism?

1 Kupffer cells
2 Eosinophils
3 Plasma cells
4 Cytotoxic T cells
5 Helper T cells
6 Natural killer cells

A

Natural killer cells

Cytotoxic T cells are CD8+ with MHC class I restriction. Direct cell contact can result in destruction of target cells without antibody involvement. Cytotoxic T cells provide primary line of defense against changed self antigen, especially virally infected cells and tumor cells. Cytotoxic T cells recognize antigen in association with MHC class I molecule, which is expressed by all nucleated host cells.
Helper T cells are CD4+ cells and recognize the antigen in conjunction with MHC class II molecule. Helper T cells are required for antibody response to most antigens. These cells control proliferation, differentiation, and isotype switching of B cells.
Natural killer cells (NK cells) are large granular lymphocytes and do not express most of the recognizable surface marker of T or B cells. NK cells can mediate cytolytic reactions and kill target cells without prior exposure to antigen. Cytolytic activity of NK cells is nonphagocytic, extracellular, and not restricted by major histocompatibility complex (MHC). NK cells are also involved in antibody dependent cell mediated cytotoxicity (ADCC).
When B cells are stimulated by an antigen, they undergo differentiation and clonal expansion. Some of the transformed B cells eventually become antibody secreting plasma cells. Plasma cells are fully differentiated lymphocytes with abundant cytoplasmic immunoglobulins. Antibody production is the primary function of plasma cells.
Kupffer cells are fixed macrophages found in liver. These cells are associated with phagocytosis and antigen presentation.
Eosinophils are homeostatic regulators of inflammation and also play a key role in the host defense against some parasites, like nematodes. Eosinophils secrete cytotoxic enzymes in their granules when attached to the surface of the parasite via IgE and damage them by way of oxidative mechanism.

69
Q

Which of the following cell types is a phagocytic antigen presenting cell found in liver cells

1 Kupffer cells
2 Eosinophils
3 Plasma cells
4 Cytotoxic T cells
5 Helper T cells
6 Natural killer cells

A

Kupffer cells

Cytotoxic T cells are CD8+ with MHC class I restriction. Direct cell contact can result in destruction of target cells without antibody involvement. Cytotoxic T cells provide primary line of defense against changed self antigen, especially virally infected cells and tumor cells. Cytotoxic T cells recognize antigen in association with MHC class I molecule, which is expressed by all nucleated host cells.
Helper T cells are CD4+ cells and recognize the antigen in conjunction with MHC class II molecule. Helper T cells are required for antibody response to most antigens. These cells control proliferation, differentiation, and isotype switching of B cells.
Natural killer cells (NK cells) are large granular lymphocytes and do not express most of the recognizable surface marker of T or B cells. NK cells can mediate cytolytic reactions and kill target cells without prior exposure to antigen. Cytolytic activity of NK cells is nonphagocytic, extracellular, and not restricted by major histocompatibility complex (MHC). NK cells are also involved in antibody dependent cell mediated cytotoxicity (ADCC).
When B cells are stimulated by an antigen, they undergo differentiation and clonal expansion. Some of the transformed B cells eventually become antibody secreting plasma cells. Plasma cells are fully differentiated lymphocytes with abundant cytoplasmic immunoglobulins. Antibody production is the primary function of plasma cells.
Kupffer cells are fixed macrophages found in liver. These cells are associated with phagocytosis and antigen presentation.
Eosinophils are homeostatic regulators of inflammation and also play a key role in the host defense against some parasites, like nematodes. Eosinophils secrete cytotoxic enzymes in their granules when attached to the surface of the parasite via IgE and damage them by way of oxidative mechanism.

70
Q

Which of the following cell types interacts with the larval stage of some helminth parasites and damages them by way of oxidative mechanism?

1 Kupffer cells
2 Eosinophils
3 Plasma cells
4 Cytotoxic T cells
5 Helper T cells
6 Natural killer cells

A

Eosinophils

ytotoxic T cells are CD8+ with MHC class I restriction. Direct cell contact can result in destruction of target cells without antibody involvement. Cytotoxic T cells provide primary line of defense against changed self antigen, especially virally infected cells and tumor cells. Cytotoxic T cells recognize antigen in association with MHC class I molecule, which is expressed by all nucleated host cells.
Helper T cells are CD4+ cells and recognize the antigen in conjunction with MHC class II molecule. Helper T cells are required for antibody response to most antigens. These cells control proliferation, differentiation, and isotype switching of B cells.
Natural killer cells (NK cells) are large granular lymphocytes and do not express most of the recognizable surface marker of T or B cells. NK cells can mediate cytolytic reactions and kill target cells without prior exposure to antigen. Cytolytic activity of NK cells is nonphagocytic, extracellular, and not restricted by major histocompatibility complex (MHC). NK cells are also involved in antibody dependent cell mediated cytotoxicity (ADCC).
When B cells are stimulated by an antigen, they undergo differentiation and clonal expansion. Some of the transformed B cells eventually become antibody secreting plasma cells. Plasma cells are fully differentiated lymphocytes with abundant cytoplasmic immunoglobulins. Antibody production is the primary function of plasma cells.
Kupffer cells are fixed macrophages found in liver. These cells are associated with phagocytosis and antigen presentation.
Eosinophils are homeostatic regulators of inflammation and also play a key role in the host defense against some parasites, like nematodes. Eosinophils secrete cytotoxic enzymes in their granules when attached to the surface of the parasite via IgE and damage them by way of oxidative mechanism.

71
Q

Which of the following cell types includes CD4+ cells involved in interaction with HLA class II antigen?

1 Kupffer cells
2 Eosinophils
3 Plasma cells
4 Cytotoxic T cells
5 Helper T cells
6 Natural killer cells

A

Helper T cells

Cytotoxic T cells are CD8+ with MHC class I restriction. Direct cell contact can result in destruction of target cells without antibody involvement. Cytotoxic T cells provide primary line of defense against changed self antigen, especially virally infected cells and tumor cells. Cytotoxic T cells recognize antigen in association with MHC class I molecule, which is expressed by all nucleated host cells.
Helper T cells are CD4+ cells and recognize the antigen in conjunction with MHC class II molecule. Helper T cells are required for antibody response to most antigens. These cells control proliferation, differentiation, and isotype switching of B cells.
Natural killer cells (NK cells) are large granular lymphocytes and do not express most of the recognizable surface marker of T or B cells. NK cells can mediate cytolytic reactions and kill target cells without prior exposure to antigen. Cytolytic activity of NK cells is nonphagocytic, extracellular, and not restricted by major histocompatibility complex (MHC). NK cells are also involved in antibody dependent cell mediated cytotoxicity (ADCC).
When B cells are stimulated by an antigen, they undergo differentiation and clonal expansion. Some of the transformed B cells eventually become antibody secreting plasma cells. Plasma cells are fully differentiated lymphocytes with abundant cytoplasmic immunoglobulins. Antibody production is the primary function of plasma cells.
Kupffer cells are fixed macrophages found in liver. These cells are associated with phagocytosis and antigen presentation.
Eosinophils are homeostatic regulators of inflammation and also play a key role in the host defense against some parasites, like nematodes. Eosinophils secrete cytotoxic enzymes in their granules when attached to the surface of the parasite via IgE and damage them by way of oxidative mechanism.

72
Q

Which of the following is true of type I hypersensitivity reaction

1 Associated with passive cutaneous anaphylaxis
2 Involves complement
3 Associated with increased production of IgG
4 Occurs after 24 hours of exposure to antigen
5 Serum sickness is type I hypersensitivity reaction

A

Associated with passive cutaneous anaphylaxis

ssive cutaneous anaphylaxis is Type I hypersensitivity reaction. This reaction is characterized by short time lag, usually seconds to minutes, between exposure to antigen and the onset of symptoms. This reaction does not involve complement. The mediators are released from mast cells and basophils. IgE production is elevated in this type of reaction.
Elevated synthesis of IgG s associated with type II and type III hypersensitivity reaction.
Serum sickness is an example of type III hypersensitivity reaction.

73
Q

A 2-year-old boy presents with audible wheezing, shortness of breath, and pruritic hives shortly after receiving the first dose of a course of amoxicillin. On examination, he is experiencing tachycardia and tachypnea and severe hypotension. He immediately receives a subcutaneous injection of epinephrine and IV diphenhydramine and methylprednisolone. His mother stated that he had taken amoxicillin for an ear infection two months earlier and had no problems.

Sensitivity to penicillin corresponds to what hypersensitivity reaction?

1 Type I hypersensitivity reaction
2 Type II hypersensitivity reaction
3 Type III hypersensitivity reaction
4 Type IV hypersensitivity reaction
5 Type V hypersensitivity reaction

A

Type I hypersensitivity reaction

Hypersensitivity to penicillin is a common cause of type 1 hypersensitivity reaction. The patient in this case is likely undergoing a type 1 (anaphylaxis) immediate hypersensitivity reaction. Penicillins are the most common cause of medication-induced anaphylaxis. Penicillin is a hapten and must bind to a carrier protein to induce a response. When penicillin is bound to its carrier protein, it can cross-link IgE and trigger the release of mediators. On repeated administration of penicillin, as in this case, the antibody can react with the hapten or the intact drug to cause rashes and an anaphylactic reaction.
Type II hypersensitivity reaction is incorrect. Type II is a cytotoxic type of reaction in which an antigen on the cell surface combines with antibody in the host and initiates complement mediated cell lysis. The clinical presentation of a type II hypersensitivity reaction is associated with transfusion reactions, hemolytic disease of the newborn, and autoimmune hemolytic anemia.
Type III hypersensitivity reaction is incorrect. In type III hypersensitivity reactions antibodies directed against soluble serum antigen form circulating complexes that deposit in tissue nonspecifically. All damage is complement mediated. Type III hypersensitivity reactions present as serum sickness, glomerulonephritis, rheumatoid arthritis, SLE, and hypersensitivity pneumonitis.
Type IV hypersensitivity reaction is incorrect. A type IV hypersensitivity reaction is a cell-mediated delayed hypersensitivity reaction due to antigen-specific TH1 cells that activate tissue macrophages and stimulate a local inflammatory response over 12-72 hours. Clinical examples include the tuberculin-type hypersensitivity (Mantoux test), contact dermatitis, granulomatous hypersensitivity, and acute tissue graft rejection.
Type V hypersensitivity reaction is incorrect. A type V classification is rarely used. This classification is reserved for receptor mediated autoimmune disease and is often used as a subcategory for type II hypersensitivity reactions. Instead of binding to cell surface components, however, the antibodies recognize and bind to cell surface receptors which either prevent ligand binding with the receptor or mimic the effect of ligand binding impairing cell signaling. Clinical examples are Graves Disease and Myasthenia Gravis neither of which are applicable in this case.

74
Q

Acute hemolytic transfusion reaction corresponds with which of the following hypersensitivity reactions?

1 Type I hypersensitivity reaction
2 Type II hypersensitivity reaction
3 Type III hypersensitivity reaction
4 Type IV hypersensitivity reaction
5 Type V hypersensitivity reaction

A

Type II hypersensitivity reaction

Serum sickness is an immune complex reaction which is a type III hypersensitivity reaction. Serum sickness develops from injection of heterologous serum protein within 1 or 2 weeks after the initial exposure. It is characterized by protracted interaction between antigen and antibody accompanied by formation of antigen-antibody complexes.
Hypersensitivity to penicillin is a common cause of type I hypersensitivity reaction. The metabolic product of penicillin acts as hapten and binds to a protein molecule and leads to antibody production and type I hypersensitivity reaction. On repeated administration of penicillin, the antibody can react with the hapten or the intact drug causing fever, rashes, and anaphylactic reaction.
Transfusion reaction is type II hypersensitivity reaction. It is a cytotoxic type of reaction in which an antigen present on cell surface combines with antibody in the host and initiates complement mediated lysis. ABO incompatibility is the most common cause of acute hemolytic transfusion reaction and occurs during or immediately after blood transfusion. Administration of erythrocytes in the presence of preexisting antibodies induce antigen-antibody reaction with an activation of complement, kinin, plasminogen and coagulation system.
Mantoux test is used to demonstrate exposure to Mycobacterium tuberculosis. Mantoux test is a delayed type of hypersensitivity reaction which is type II hypersensitivity reaction. Development of erythema and the induration at the site of injection indicates previous exposure or sensitization with the antigen. This type of reaction starts hours or days after contact with the antigen and consists mainly of macrophages and helper T lymphocytes.

75
Q

Tuberculin test corresponds with which of the following hypersensitivy reactions?

1 Type I hypersensitivity reaction
2 Type II hypersensitivity reaction
3 Type III hypersensitivity reaction
4 Type IV hypersensitivity reaction
5 Type V hypersensitivity reaction

A

Type IV hypersensitivity reaction

Serum sickness is an immune complex reaction which is a type III hypersensitivity reaction. Serum sickness develops from injection of heterologous serum protein within 1 or 2 weeks after the initial exposure. It is characterized by protracted interaction between antigen and antibody accompanied by formation of antigen-antibody complexes.
Hypersensitivity to penicillin is a common cause of type I hypersensitivity reaction. The metabolic product of penicillin acts as hapten and binds to a protein molecule and leads to antibody production and type I hypersensitivity reaction. On repeated administration of penicillin, the antibody can react with the hapten or the intact drug causing fever, rashes, and anaphylactic reaction.
Transfusion reaction is type II hypersensitivity reaction. It is a cytotoxic type of reaction in which an antigen present on cell surface combines with antibody in the host and initiates complement mediated lysis. ABO incompatibility is the most common cause of acute hemolytic transfusion reaction and occurs during or immediately after blood transfusion. Administration of erythrocytes in the presence of preexisting antibodies induce antigen-antibody reaction with an activation of complement, kinin, plasminogen and coagulation system.
Mantoux test is used to demonstrate exposure to Mycobacterium tuberculosis. Mantoux test is a delayed type of hypersensitivity reaction which is type IV hypersensitivity reaction. Development of erythema and the induration at the site of injection indicates previous exposure or sensitization with the antigen. This type of reaction starts hours or days after contact with the antigen and consists mainly of macrophages and helper T lymphocytes.

76
Q

Which of the following is true of infections caused by human immunodeficiency virus?

1 CD8 to CD4 ratio is reduced
2 Does not infect monocytes
3 P24 protein on the viral envelope binds to CD10 protein receptor on target cell
4 Causes functional inactivation and lysis of helper T cells
5 Can not be acquired via sexual contact and blood products

A

Causes functional inactivation and lysis of helper T cells

Human immunodeficiency virus (HIV) is associated acquired immunodeficiency syndrome (AIDS) in human. The virus primarily infects CD4 positive cells. GP120 protein on the viral envelope binds to the CD4 protein receptor on the target cells. These cells include helper T cells, monocytes, macrophages, microglial brain cells, and B cells.
HIV can be transmitted via sexual contact and infected blood products. HIV infection results in functional inactivation and lysis of helper T cells. This leads to decrease in CD4+ cells and reduced CD4 to CD8 ratio (less than 1.0).
AIDS patients are highly susceptible to opportunistic infection.

77
Q

Which of the following is an organ-specific autoimmune disease affecting thyroid?

1 Graves’ disease
2 Addison’s disease
3 Autoimmune hemolytic anemia
4 Goodpasture’s syndrome
5 Rheumatoid arthritis

A

Graves’ disease

Graves’ disease and Hashimoto’s thyroiditis are organ-specific autoimmune diseases affecting thyroid. Unlike Hashimoto’s thyroiditis, Graves’ disease causes hyperthyroidism. Graves’ disease is characterized by soft enlarged goiter and exophthalmus, which gives “large eyed staring expression” to the patient. Thyroid stimulating hormone receptor antibody is most significant autoantibody found in Graves’ disease.
Pernicious anemia results from antibodies directed against intrinsic factors and parietal cells in the stomach.
Autoimmune hemolytic anemia are caused by antibodies directed to red blood cells
Goodpasture’s syndrome affects kidneys and lungs. It results from glomerulonephritis due to antibodies directed against kidney cells.
Rheumatoid arthritis is systemic autoimmune disorder and involves synovial membrane of joints. Autoantibodies associated with rheumatoid arthritis include rheumatoid factor, antinuclear antibodies, and anticollagen antibodies.

78
Q

Kidney transplant between identical sisters is an example of

1 Autograft
2 Allograft
3 Xenograft
4 Syngraft

A

Syngraft

Kidney transplant between genetically identical sisters is an example of syngraft. In this type of transplant, grafted tissue or organ contains similar antigenic determinants.
Autograft involves tissue transplant (such as skin or bone marrow) from one position to another in the same individual. This type of graft involves no difference in antigenic determinants.
Allograft is graft exchange between genetically different recipient and donor of the same species. Antigenic compatibility must be tested in allograft as the graft contains antigen not present in the recipient.
Xenograft is a graft exchange between recipient and donor of the different species and mostly involves extensive immunologic differences.

79
Q

The oncofetal antigen used as tumor marker in monitoring and diagnosis of germ cell tumor and liver tumor is

1 CA 15-3
2 CA 125
3 Carcinoembryonic antigen (CEA)
4 Alpha-fetoprotein (AFP)
5 Prostatic acid phosphatase

A

Alpha-fetoprotein (AFP)

Tumor marker is a characteristic of a neoplastic cell and can be measured in tissues and body fluids using immunochemical assays. Detection of tumor marker is helpful in finding location of tumor. Quantitative determination of particular tumor marker is useful in monitoring the patient’s response to therapy.
Alpha-fetoprotein (AFP) is an oncofetal antigen which is normally secreted by fetal liver cells and yolk sac cells. AFP is not present in detectable levels after birth in normal individuals. AFP is a tumor associated antigen which is re-expressed by rapidly dividing cells or neoplastic cells.
Elevated levels of AFP are associated with liver tumors and germ cell tumors. AFP is a reliable tumor marker for monitoring patient’s response to chemotherapy and radiation therapy.
CA 15-3 is a tumor marker used in detection of breast cancer. It is also used for monitoring therapy and recurrence of breast cancer.
CA 125 is an oncofetal antigen used in diagnosis and monitoring of ovarian cancer.
Carcinoembryonic antigen (CEA) is associated with various types of cancer. CEA is of diagnostic value in monitoring patients with colon cancer.
Prostatic acid phosphatase is an enzyme used for diagnosis of prostatic carcinoma.

80
Q

Immunoglobulin replacement therapy is critical for children and adults with defective antibody production. Intravenous and intramuscular immunoglobulin preparations are available, and the choice depends on the severity of the hypogammaglobulinemia and pre-existing complications. For most patients, intravenous immunoglobulin (IVIG) is the treatment of choice. IVIG is derived from a plasma pool consisting of several thousand donor units in order to give the widest possible range of protective antibodies. What technique is used for plasma fractionation in the preparation of IVIG?

1 Enzyme Linked ImmunoSorbent Assay (ELISA)
2 Polymerase Chain Reaction (PCR)
3 Gel Electrophoresis
4 Flourescence-Activated Cell Sorter (FACS)
5 Cohn-Oncley method

A

Cohn-Oncley method

Plasma fractionation is carried out by the Cohn-Oncley cold ethanol method. Some 60 years ago, Cohn and his colleagues devised a method for the separation of the gamma globulin fraction from human serum by precipitation with cold ethanol. This “Cohn fractionation” represented a practical and safe method for production of homologous human antibody for clinical use. Plasma is collected from healthy donors. Large quantities can be obtained by plasmapheresis. Currently, modified Cohn or the Cohn-Oncley fractionation method is followed, in which plasma proteins are precipitated by a combination of cold ethanol-water mixtures and adjustments of pH, ionic strength, temperature, and protein concentrations. This is followed by various other treatments, including fractionation using polyethylene glycol or ion exchange chromatography; acidification to pH 4-4.5; exposure to pepsin or trypsin; and stabilization with maltose, sucrose, glucose, or glycine. This stabilization reduces aggregation of the globulins that can trigger anaphylactoid reactions. In the Cohn-Oncley method, Fraction II and III have a high concentration of immunoglobulins.
Enzyme-linked immunosorbent assay, or ELISA, is a direct binding assay to detect the presence of a specific antibody or to measure its quantity in a sample. An antigen or a specific antibody can be detected by a reaction that converts a colorless substrate into a colored reaction product.
PCR, or polymerase chain reaction, is a method for dramatic amplification of target DNA prior to cleavage with a restriction enzyme. This powerful and sensitive technique can detect a specific DNA sequence from a single cell, fixed pathological specimens, and dried blood spots.
Gel electrophoresis is a method involved in the separation of macromolecules like nucleic acids, or proteins on the basis of size, electrical charge, and other physical properties. Trace DNA samples obtained from blood drop or semen can be subjected to gel electrophoresis for DNA separation.
By examining large numbers of cells, the fluorescence-activated cell sorter, or FACS, can give quantitative data on the percentage of cells bearing different molecules, such as surface immunoglobulin. The flow cytometer detects and counts individual cells passing in a stream through a laser beam. Cell subsets are identified using labeled monoclonal antibodies to cell-surface proteins.

81
Q

Which of the following describes the immunostimulatory effect of cimetidine?

1 Cimetidine blocks H2 receptors on B cells, diminishing the humoral response, thus increasing cellular immunity
2 Cimetidine blocks H2 receptors on B cells, decreasing their cAMP levels, increasing their activity
3 Cimetidine blocks H2 receptors on T cells, decreasing their cAMP levels, increasing their activity
4 Cimetidine stimulates H1 receptors on T cells, increasing their cAMP levels, increasing their activity
5 Cimetidine stimulates H1 receptors on B cells, increasing their cAMP levels, increasing their activity

A

Cimetidine blocks H2 receptors on T cells, decreasing their cAMP levels, increasing their activity

T cells have both H1 and H2 receptors on their surface. Blocking of H2 receptors by cimetidine depresses intracellular levels of cAMP, increasing the activity of T cells.

82
Q

You are treating a 57 year-old female for rheumatoid arthritis. Your patient, by her own reckoning, is not making satisfactory progress with conventional therapy of corticosteroids and NSAIDs. You fancy yourself as a maverick immunotherapist and decide to try some nonconventional treatments to your case. What is the mechanism by which antimalarial compounds may help your patient

1 Chloroquine compounds interfere with antigen processing by antigen presenting cells
2 Chloroquine compounds inhibit T celll activation by blocking CD 28
3 Chloroquine compounds inhibit prostaglandin synthesis as well as phagocytosis and chemotaxis
4 Chloroquine compounds damage T cells by enhancing local hydrogen peroxide formation
5 Chloroquine compounds inhibit synthesis of IL-2

A

Chloroquine compounds interfere with antigen processing by antigen presenting cells

Chloroquine compounds enter lysosomes and other inclusions in which antigen presentation takes place. Chloroquine raises the pH of the lysosomes, inhibiting the enzymatic activity of the proteases required for antigen processing for antigen presentation.

83
Q

You are treating a 57-year-old female for rheumatoid arthritis. Your patient, by her own reckoning, is not making satisfactory progress with conventional therapy of corticosteroids and NSAIDs. You fancy yourself as a maverick immunotherapist and decide to try some nonconventional treatments. If you decided to try D-penicillamine to treat your patient’s RA, by which mechanism would you predict that this compound would work?

1 D-penicillamine interferes with antigen processing by antigen presenting cells
2 D-penicillamine inhibits T cell activation by blocking CD 28
3 D-penicillamine inhibits prostaglandin synthesis as well as phagocytosis and chemotaxis
4 D-penicillamine damages T cells by enhancing local hydrogen peroxide formation
5 D-penicillamine inhibits synthesis of IL-2

A

D-penicillamine damages T cells by enhancing local hydrogen peroxide formation

It is thought that when in the presence of copper salts, D-penicillamine initiates hydrogen peroxide production, damaging the T cells.

84
Q

You are treating a 57 year-old female for rheumatoid arthritis. Your patient, by her own reckoning, is not making satisfactory progress with conventional therapy of corticosteroids and NSAIDs. You fancy yourself as a maverick immunotherapist and decide to try some nonconventional treatments to your case. You decide to go out on a limb and try cyclosporine A as a treatment for rheumatoid arthritis. By what mechanism does CyA work?

1 Cyclosporine A interferes with antigen processing by antigen presenting cells
2 Cyclosporine A inhibits T cell activation by blocking CD 28
3 Cyclosporine A inhibits prostaglandin synthesis as well as phagocytosis and chemotaxis
4 Cyclosporine A damages T cells by enhancing local hydrogen peroxide formation
5 Cyclosporine A inhibits synthesis of IL-2

A

Cyclosporine A inhibits synthesis of IL-2

CyA mainly inhibits synthesis of IL-2, inhibiting IL-2-dependant activation of T cells. However, treatment of RA patients with CyA has had only marginal success.

85
Q

You have a 48-year-old female with hereditary angioedema. Your patient lacks which member of the complement system?

1 C1s
2 C1q
3 C2
4 C1-INH
5 C3

A

C1-INH

C1 Inhibitor (C1-INH) controls the classical pathway by binding and blocking the activity of C1r and C1s. In hereditary angioedema, excessive amounts of C2 kinin are produced, which increases the vascular permeability.

86
Q

Your patient, a 37 year-old Caucasian female, presents to you with systemic lupus erythematosus (SLE). She is suffering from glomerulonephritis and rheumatoid disease. As far as the complement system is concerned, which of the following is your patient most likely deficient?

1 C3
2 C4a and C4b
3 C1-INH
4 C6 and C7
5 C9

A

C4a and C4b

90% of individuals lacking C4a and C4b develop SLE. In their absence, C3 does not bind immune complexes. Lack of C3 binding of immune complexes results in decrease clearance of the complexes from the blood and subsequent deposition throughout the body. This initiates some of the symptoms associated with SLE.

87
Q

Which one of the following immunoglobulins is most abundant in normal human serum?

1 IgG
2 IgM
3 IgA
4 IgD
5 IgE

A

IgG

IgG is the most abundant immunoglobulin in normal human serum as it constitutes about 75% of the total immunoglobulin. The abundance of the other immunoglobulins is as follows: IgM-10%, IgA-15%, IgD-trace, and IgE-minute.

88
Q

Which one of the following immunoglobulins has the longest half life in serum?

1 IgG
2 IgM
3 IgA
4 IgD
5 IgE

A

IgG

IgG has the longest half life of 23 days in serum. The half lives of other immunoglobulins are as follows: IgM-5 days, IgA-6 days, IgD-3 days, and IgE-2.5 days.

89
Q

When T lymphocytes carrying specific antigen receptors become activated by contact with that antigen they release lymphokines, which in turn act on the macrophages, lymphocytes, and other cells. They in turn yield which one of the following types of delayed hypersensitivity?

1 Anaphylactic type
2 Cytotoxic antigens
3 Complex-mediated antigen antibody
4 Cell-mediated
5 Stimulatory

A

Cell-mediated

When T lymphocytes carrying specific antigen receptors become activated by contact with that antigen they release lymphokines which, in turn, act on the macrophages, lymphocytes, and other cells to yield the cell-mediated hypersensitivity. Anaphylactic hypersensitivity occurs when a special class of antibody (cytotropic antibody, mainly IgE) binds to mast cells and basophils through the Fc fragment. When an antigen reacts with these antibodies, vasoactive amines and other mediators are released and elicit the reaction. Cytotoxic type of hypersensitivity occurs when antigens on the surface combine with an antibody. This may lead to opsonization and phagocytosis without complement, may facilitate an attack by killer cells, or may lead to binding of complement. This promotes immune adherence to phagocytes or the lytic effect, which may result in membrane damage by the complement. Stimulatory hypersensitivity occurs when non-complement binding antibody against cell surface components may actually stimulate the cell.

90
Q

The smallest unit of a complex antigen capable of binding to antibody is known as

1 Immunogen
2 Epitope
3 Hapten
4 Adjuvant
5 Carrier molecule

A

Epitope

Epitopes are an integral part of an antigen molecule that binds with antibody. Epitopes are also known as antigenic determinant as they determine the specificity of the antigen molecule. Epitopes can exist in sequential form or conformational form.
Haptens are small molecules that bind to carrier molecules and become immunogenic.
Adjuvants are substances that enhance the immunogenicity of molecules without changing their chemical structure.

91
Q

Antigens that are found in genetically different members of the same species are called

1 Heterologous antigens
2 Autologous antigens
3 Syngeneic antigens
4 Allogeneic antigens
5 Xenogeneic antigens

A

Allogeneic antigens

Allogeneic antigens are found in genetically different members of one’s own species.
Heterologous antigens are also called xenogeneic antigens and are present in different species.
Autologous antigens are self antigens found in an individual.
Syngeneic antigens are found in genetically identical individuals.

92
Q

Which of the following antibodies is found in the highest concentration in serum?

1 IgA
2 IgD
3 IgE
4 IgG
5 IgM

A

IgG

IgG is the predominating antibody in serum and contributes to 75% - 85% of total immunoglobulin concentration. IgG is responsible for protection of the fetus, as it is the only immunoglobulin that can cross the placenta. It is a divalent molecule and is made up of 2 H chains and 2 lights chain. IgG is divided into 4 subclasses: IgG1, IgG2, IgG3, and IgG4, according to antigenic differences in the H chain and the number and location of disulfide bonds. IgG is also involved in Rh disease of the newborn. It is produced in high concentration during secondary immune response.
IgM can fix the complement but cannot cross the placenta. It is the largest immunoglobulin and is a pentamer made up of 5 H2L2 units and 1 molecule of J chain. It has 10 antigen binding sites and has the highest avidity. It is the first antibody to appear in an immune response.
IgA is produced by B cells and is found in high concentrations in tears, saliva, milk, and other secretions. It is a monomer made up of 1 4-chain unit.
IgE is a monomer that is present in very low concentration in serum. An increased level of IgE is found in antibody mediated allergic hypersensitivity and in parasitic infections. The mast cells and the basophils are involved in allergic response and have Fc receptors specific for IgE antibodies. The IgE antibodies serve as antigen receptors and can cause mast cells and basophils to release inflammatory mediators like histamine and heparin.
Ig D is a monomer comprised of 4-chain units and is found on the surface of B lymphocytes. It is found in very low concentration in serum.
IgA, IgE, and IgD do not fix the complement and cannot cross the placenta.

93
Q

The antibody that can cross the placenta and can fix the complement is

1 IgA
2 IgD
3 IgE
4 IgG
5 IgM

A

IgG

IgG is the predominating antibody in serum and contributes to 75% - 85% of total immunoglobulin concentration. IgG is responsible for protection of the fetus, as it is the only immunoglobulin that can cross the placenta. It is a divalent molecule and is made up of 2 H chains and 2 lights chain. IgG is divided into 4 subclasses: IgG1, IgG2, IgG3, and IgG4, according to antigenic differences in the H chain and the number and location of disulfide bonds. IgG is also involved in Rh disease of the newborn. It is produced in high concentration during secondary immune response.
IgM can fix the complement but cannot cross the placenta. It is the largest immunoglobulin and is a pentamer made up of 5 H2L2 units and 1 molecule of J chain. It has 10 antigen binding sites and has the highest avidity. It is the first antibody to appear in an immune response.
IgA is produced by B cells and is found in high concentrations in tears, saliva, milk, and other secretions. It is a monomer made up of 1 4-chain unit.
IgE is a monomer that is present in very low concentration in serum. An increased level of IgE is found in antibody mediated allergic hypersensitivity and in parasitic infections. The mast cells and the basophils are involved in allergic response and have Fc receptors specific for IgE antibodies. The IgE antibodies serve as antigen receptors and can cause mast cells and basophils to release inflammatory mediators like histamine and heparin.
Ig D is a monomer comprised of 4-chain units and is found on the surface of B lymphocytes. It is found in very low concentration in serum.
IgA, IgE, and IgD do not fix the complement and cannot cross the placenta.

94
Q

The antibody that is a pentamer and is first to appear in an immune response is:

1 IgA
2 IgD
3 IgE
4 IgG
5 IgM

A

IgM

IgG is the predominating antibody in serum and contributes to 75% - 85% of total immunoglobulin concentration. IgG is responsible for protection of the fetus, as it is the only immunoglobulin that can cross the placenta. It is a divalent molecule and is made up of 2 H chains and 2 lights chain. IgG is divided into 4 subclasses: IgG1, IgG2, IgG3, and IgG4, according to antigenic differences in the H chain and the number and location of disulfide bonds. IgG is also involved in Rh disease of the newborn. It is produced in high concentration during secondary immune response.
IgM can fix the complement but cannot cross the placenta. It is the largest immunoglobulin and is a pentamer made up of 5 H2L2 units and 1 molecule of J chain. It has 10 antigen binding sites and has the highest avidity. It is the first antibody to appear in an immune response.
IgA is produced by B cells and is found in high concentrations in tears, saliva, milk, and other secretions. It is a monomer made up of 1 4-chain unit.
IgE is a monomer that is present in very low concentration in serum. An increased level of IgE is found in antibody mediated allergic hypersensitivity and in parasitic infections. The mast cells and the basophils are involved in allergic response and have Fc receptors specific for IgE antibodies. The IgE antibodies serve as antigen receptors and can cause mast cells and basophils to release inflammatory mediators like histamine and heparin.
Ig D is a monomer comprised of 4-chain units and is found on the surface of B lymphocytes. It is found in very low concentration in serum.
IgA, IgE, and IgD do not fix the complement and cannot cross the placenta.

95
Q

The antibody with 10 antigen binding sites is

1 IgA
2 IgD
3 IgE
4 IgG
5 IgM

A

IgM

IgG is the predominating antibody in serum and contributes to 75% - 85% of total immunoglobulin concentration. IgG is responsible for protection of the fetus, as it is the only immunoglobulin that can cross the placenta. It is a divalent molecule and is made up of 2 H chains and 2 lights chain. IgG is divided into 4 subclasses: IgG1, IgG2, IgG3, and IgG4, according to antigenic differences in the H chain and the number and location of disulfide bonds. IgG is also involved in Rh disease of the newborn. It is produced in high concentration during secondary immune response.
IgM can fix the complement but cannot cross the placenta. It is the largest immunoglobulin and is a pentamer made up of 5 H2L2 units and 1 molecule of J chain. It has 10 antigen binding sites and has the highest avidity. It is the first antibody to appear in an immune response.
IgA is produced by B cells and is found in high concentrations in tears, saliva, milk, and other secretions. It is a monomer made up of 1 4-chain unit.
IgE is a monomer that is present in very low concentration in serum. An increased level of IgE is found in antibody mediated allergic hypersensitivity and in parasitic infections. The mast cells and the basophils are involved in allergic response and have Fc receptors specific for IgE antibodies. The IgE antibodies serve as antigen receptors and can cause mast cells and basophils to release inflammatory mediators like histamine and heparin.
Ig D is a monomer comprised of 4-chain units and is found on the surface of B lymphocytes. It is found in very low concentration in serum.
IgA, IgE, and IgD do not fix the complement and cannot cross the placenta.

96
Q

The antibody predominant in milk, saliva, and tears is

1 IgA
2 IgD
3 IgE
4 IgG
5 IgM

A

IgA

IgG is the predominating antibody in serum and contributes to 75% - 85% of total immunoglobulin concentration. IgG is responsible for protection of the fetus, as it is the only immunoglobulin that can cross the placenta. It is a divalent molecule and is made up of 2 H chains and 2 lights chain. IgG is divided into 4 subclasses: IgG1, IgG2, IgG3, and IgG4, according to antigenic differences in the H chain and the number and location of disulfide bonds. IgG is also involved in Rh disease of the newborn. It is produced in high concentration during secondary immune response.
IgM can fix the complement but cannot cross the placenta. It is the largest immunoglobulin and is a pentamer made up of 5 H2L2 units and 1 molecule of J chain. It has 10 antigen binding sites and has the highest avidity. It is the first antibody to appear in an immune response.
IgA is produced by B cells and is found in high concentrations in tears, saliva, milk, and other secretions. It is a monomer made up of 1 4-chain unit.
IgE is a monomer that is present in very low concentration in serum. An increased level of IgE is found in antibody mediated allergic hypersensitivity and in parasitic infections. The mast cells and the basophils are involved in allergic response and have Fc receptors specific for IgE antibodies. The IgE antibodies serve as antigen receptors and can cause mast cells and basophils to release inflammatory mediators like histamine and heparin.
Ig D is a monomer comprised of 4-chain units and is found on the surface of B lymphocytes. It is found in very low concentration in serum.
IgA, IgE, and IgD do not fix the complement and cannot cross the placenta.

97
Q

The antibody responsible for mucous membrane protection from bacterial and virus attack is

1 IgA
2 IgD
3 IgE
4 IgG
5 IgM

A

IgA

IgG is the predominating antibody in serum and contributes to 75% - 85% of total immunoglobulin concentration. IgG is responsible for protection of the fetus, as it is the only immunoglobulin that can cross the placenta. It is a divalent molecule and is made up of 2 H chains and 2 lights chain. IgG is divided into 4 subclasses: IgG1, IgG2, IgG3, and IgG4, according to antigenic differences in the H chain and the number and location of disulfide bonds. IgG is also involved in Rh disease of the newborn. It is produced in high concentration during secondary immune response.
IgM can fix the complement but cannot cross the placenta. It is the largest immunoglobulin and is a pentamer made up of 5 H2L2 units and 1 molecule of J chain. It has 10 antigen binding sites and has the highest avidity. It is the first antibody to appear in an immune response.
IgA is produced by B cells and is found in high concentrations in tears, saliva, milk, and other secretions. It is a monomer made up of 1 4-chain unit.
IgE is a monomer that is present in very low concentration in serum. An increased level of IgE is found in antibody mediated allergic hypersensitivity and in parasitic infections. The mast cells and the basophils are involved in allergic response and have Fc receptors specific for IgE antibodies. The IgE antibodies serve as antigen receptors and can cause mast cells and basophils to release inflammatory mediators like histamine and heparin.
Ig D is a monomer comprised of 4-chain units and is found on the surface of B lymphocytes. It is found in very low concentration in serum.
IgA, IgE, and IgD do not fix the complement and cannot cross the placenta.

98
Q

A high level of this antibody is found in persons with antibody mediated allergic hypersensitivity.

1 IgA
2 IgD
3 IgE
4 IgG
5 IgM

A

IgE

IgG is the predominating antibody in serum and contributes to 75% - 85% of total immunoglobulin concentration. IgG is responsible for protection of the fetus, as it is the only immunoglobulin that can cross the placenta. It is a divalent molecule and is made up of 2 H chains and 2 lights chain. IgG is divided into 4 subclasses: IgG1, IgG2, IgG3, and IgG4, according to antigenic differences in the H chain and the number and location of disulfide bonds. IgG is also involved in Rh disease of the newborn. It is produced in high concentration during secondary immune response.
IgM can fix the complement but cannot cross the placenta. It is the largest immunoglobulin and is a pentamer made up of 5 H2L2 units and 1 molecule of J chain. It has 10 antigen binding sites and has the highest avidity. It is the first antibody to appear in an immune response.
IgA is produced by B cells and is found in high concentrations in tears, saliva, milk, and other secretions. It is a monomer made up of 1 4-chain unit.
IgE is a monomer that is present in very low concentration in serum. An increased level of IgE is found in antibody mediated allergic hypersensitivity and in parasitic infections. The mast cells and the basophils are involved in allergic response and have Fc receptors specific for IgE antibodies. The IgE antibodies serve as antigen receptors and can cause mast cells and basophils to release inflammatory mediators like histamine and heparin.
Ig D is a monomer comprised of 4-chain units and is found on the surface of B lymphocytes. It is found in very low concentration in serum.
IgA, IgE, and IgD do not fix the complement and cannot cross the placenta.

99
Q

The antibody which mediates the release of histamine and heparin is

1 IgA
2 IgD
3 IgE
4 IgG
5 IgM

A

IgE

IgG is the predominating antibody in serum and contributes to 75% - 85% of total immunoglobulin concentration. IgG is responsible for protection of the fetus, as it is the only immunoglobulin that can cross the placenta. It is a divalent molecule and is made up of 2 H chains and 2 lights chain. IgG is divided into 4 subclasses: IgG1, IgG2, IgG3, and IgG4, according to antigenic differences in the H chain and the number and location of disulfide bonds. IgG is also involved in Rh disease of the newborn. It is produced in high concentration during secondary immune response.
IgM can fix the complement but cannot cross the placenta. It is the largest immunoglobulin and is a pentamer made up of 5 H2L2 units and 1 molecule of J chain. It has 10 antigen binding sites and has the highest avidity. It is the first antibody to appear in an immune response.
IgA is produced by B cells and is found in high concentrations in tears, saliva, milk, and other secretions. It is a monomer made up of 1 4-chain unit.
IgE is a monomer that is present in very low concentration in serum. An increased level of IgE is found in antibody mediated allergic hypersensitivity and in parasitic infections. The mast cells and the basophils are involved in allergic response and have Fc receptors specific for IgE antibodies. The IgE antibodies serve as antigen receptors and can cause mast cells and basophils to release inflammatory mediators like histamine and heparin.
Ig D is a monomer comprised of 4-chain units and is found on the surface of B lymphocytes. It is found in very low concentration in serum.
IgA, IgE, and IgD do not fix the complement and cannot cross the placenta.

100
Q

IgM

1 Is a pentamer
2 Has 5 antigen binding site
3 Can cross the placenta
4 Is predominant in secretion
5 Does not participate in agglutination reaction

A

Is a pentamer

IgM is a pentamer made up of 5 H2L2 units and 1 molecule of J chain. It has 10 antigen binding sites and is an excellent agglutinating antibody. It is the first antibody to appear in an immune response. IgM can fix the complement and can kill the bacterial cell by cytolysis. IgM is the largest immunoglobulin and cannot cross the placenta. The predominating antibody in the secretions is IgA.

101
Q

Select the statement that describes the antibody structure correctly

1 Light chain that is different to all class of antibody
2 Heavy chain is common for each class and subclass
3 Light chain has one variable and one constant region
4 Fc fragment is on amino terminal
5 Antigen binding site is present on carboxy terminal of the antibody molecule

A

Light chain has one variable and one constant region

The basic unit of an antibody is made up of two light chain and two heavy chains.
An equal number of heavy and light chain polypeptides are present in every immunoglobulin molecule and can be represented by the formula:- (H2L2)n. The polypeptide chains are linked covalently by disulfide bonds. Additional characteristics of structure of an antibody molecule are as follows:
Consists of light chain that is common to all class
Heavy chain is different for each class and subclass
Light chain has one variable and one constant region
Fc fragment is on carboxy terminal
Antigen binding site is present on the N terminal of the antibody molecule, which is the extremity terminated by an amino acid with a free amine group (NH2).

102
Q

Which of the following forms an antigen binding site in an immunoglobulin molecule?

1 Amino terminal of each light chain of constant region
2 Carboxy terminal of constant heavy chain
3 Amino terminal of variable light and heavy chain
4 Carboxy terminal of variable heavy chain
5 Amino terminal of constant heavy and constant light chain

A

Amino terminal of variable light and heavy chain

Variable region of light chain and heavy chains consists of N-terminal that shows variation in amino acid sequences. Amino terminal of this region forms an antigen binding site.
Constant region consists of carboxy terminal and does not demonstrate variation in amino acid sequences. This region is responsible for biological functions.

103
Q

Ataxia telangiectasia is

1 An X-linked recessive syndrome
2 A combined defect of both cellular and humoral immunity
3 Characterized by elevated IgA levels
4 Associated with defective neutrophil function
5 An acquired C3 deficiency

A

A combined defect of both cellular and humoral immunity

Ataxia telangiectasia is an autosomal recessive syndrome characterized by a combined defect of both cellular and humoral immunities. Cerebellar ataxia (staggering) and telangiectasia (enlargement of blood vessels), especially affecting earlobe and conjunctiva, are seen in this syndrome. Recurrent infections occur as antibody response to antigen is inhibited. Patients suffering from ataxia telangiectasia often demonstrate reduced levels of IgA, IgE, and IgG2, and a decrease in circulating T lymphocytes.

104
Q

Which of the following cells can retain the memory of prior contact with an antigen?

1 B lymphocytes
2 Macrophages
3 Basophils
4 Thrombocytes
5 Neutrophils

A

B lymphocytes

Lymphocytes are the only immunologically specific cellular component of the immune system. Lymphocytes are responsible for primary recognition of an antigen. Antigen binding to B lymphocyte results in the enlargement, proliferation, and differentiation of lymphocytes. Some of the proliferating cells do not differentiate into plasma cells but instead revert to a resting state to become memory B lymphocytes. These memory cells survive for years within lymphoid follicles and produce more memory and effector cells upon activation by antigens.
Macrophages are involved in phagocytosis, antigen presentation, and induction of immune response
Basophils consist of heparin and histamine in their granules and participate in acute and systemic hypersensitivity reactions
Thrombocytes are also known as platelet and participate in the clotting mechanism of the body
Neutrophils are the principle leukocytes associated with phagocytosis and localized inflammatory response

105
Q

Individuals of blood group type O

1 Have O antigen on their red blood cells
2 Have A antigen and B antigen on their red blood cells
3 Have neither A nor B antigen on their red blood cells
4 Lack anti-A and anti-B antibodies
5 Lack anti-A antibody

A

Have neither A nor B antigen on their red blood cells

Blood group antigens are found on red blood cells in the blood. Antibodies to specific blood groups are found in all individuals that lack that antigen. Type O individuals lack A and B antigens; therefore, they have both anti-A and anti-B antibodies.
Hemagglutination reaction is used to type blood groups and match compatibility between donors and recipients for blood transfusion.
Type AB individuals have A antigen and B antigen on their red blood cells and therefore lack anti-A and anti-B antibodies.
Type A individuals have A antigens on their red blood cells and therefore lack anti-A antibody and contain anti-B antibody.
Type B individuals have B antigens on their red blood cells and therefore lack anti-B antibody and contain anti-A antibody.

106
Q

Which of the following immunologic reactions involves the combination of soluble antigen with soluble antibody to produce insoluble complexes?

1 Complement fixation
2 Agglutination
3 Immunofluorescence
4 Precipitation
5 Flocculation

A

Precipitation

Precipitation involves the interaction of soluble antigen with a soluble antibody in correct proportion, leading to a visible precipitate. Precipitation reaction is affected by the amount of antigen and antibody present. The largest amount of precipitate is found in zone of equivalence.
Complement fixation is the triggering of the classical complement pathway resulting from the combination of antigen with specific antibody.
Agglutination reaction results when particulate antigens aggregate to form larger complexes in the presence of specific antibody.
Flocculation is an antigen-antibody reaction, where antigen is an insoluble particulate.
Immunofluorescence involves the use of fluorescence microscope and is used for antigen detection in cells or tissues through the use of specific antibody, labeled with fluorescent compound.

107
Q

Which of the following tests is used for confirmation of HIV infection?

1 Western blot assay
2 Enzyme linked immunosorbent assay
3 PPD test
4 RPR test
5 Rose-Waaler test

A

Western blot assay

Enzyme linked immunosorbent assay (ELISA) is used as a screening test for HIV infection. When results for ELISA for antibody to HIV is positive, western blot is used to determine if the results are true positive or false positive.
Western blot assay is used as a confirmatory test to detect antibodies to HIV-1, human immunodeficiency virus. Western blot is an adaptation of immunofixation electrophoresis and is used to detect the antibodies to organisms of complex antigenic composition. HIV antigens are electrophoresed in a gel resulting in separation of individual viral proteins. These viral proteins are then transferred to nitrocellulose paper by means of blotting, and the patient’s serum is applied over it. If antibodies are present, they bind to viral proteins and can be detected by addition of antibody to human IgG labeled with either radioactive material or an enzyme which produces a visible color change.
Rose-Waller test is used for detection of rheumatoid factor.
PPD test or purified protein derivative test is a skin test available to detect sensitization with Mycobacterium tuberculosis.
RPR test or rapid plasma reagin test is a screening test used for diagnosis of syphilis.

108
Q

Which of the following methods is most sensitive for measurements of antigen and antibody?

1 Precipitation
2 Indirect enzyme linked immunosorbent assay
3 Bacterial agglutination
4 Immunoelectrophoresis
5 Passive hemagglutination

A

Indirect enzyme linked immunosorbent assay

Indirect or noncompetitive enzyme linked immunosorbent assay (ELISA) has higher sensitivity (0.0005μg/mL) and is used for qualitative and quantitative determination of unknown antibodies in a patient’s specimen.
In this method, an antigen is bound to solid phase, and unlabeled patient antibody is added and given time to react. Following the wash step, an enzyme labeled antiglobulin is added. The second labeled antibody reacts with the Fc portion of patient’s antibody that is bound to solid phase. In absence of a specific antibody, there is no binding of second labeled antibody. In the wash step, all unbound labeled antibody is removed. In the next step, substrate is added and color development is measured. Color development is directly proportional to the amount of antibody in a patient’s serum.
Indirect ELISA is more sensitive, and sensitivity of nanogram is achieved because all the patient’s ligand has a chance to participate in the reaction.
Sensitivity for precipitation and immunoelectrophoresis method is ~20μg/mL for antibody detection
Sensitivity for bacterial agglutination and passive hemagglutination methods is ~0.01μg/mL for antibody detection.

109
Q

3 basic reaction patterns result from the relationship of antigen and antibody in Ouchterlony double diffusion technique. The appropriate observation for partial identity is

1 Lines of precipitation run parallel to each other
2 Lines of precipitation merge with spur formation
3 Lines of precipitation coming together forming a smooth curve
4 Lines of precipitation cross each other
5 Lines of precipitation form 2 concentric circles

A

Lines of precipitation merge with spur formation

In Ouchterlony double diffusion technique, both antigen and antibody diffuse independently through the medium in vertical and horizontal dimensions. In this method, wells are cut into an agar plate and antigens and antisera are placed in the wells. The diffusion results in formation of a visible precipitation band at the zone of equivalence. When antibodies are placed in the center well and antigens are placed in the surrounding wells, 3 basic reactions are possible depending on the relationship between the antigens and antibody.
Reaction of partial identity is indicated when precipitation line fuses with spur formation. This reaction occurs when antigens being tested are nonidentical but do share common determinants.
Reaction of identity is revealed when the precipitation lines meet at a point and form a smooth curve or arc. This type of precipition band indicates that antibody is precipitating identical antigen specificities in each preparation. Lines of identity do not necessarily mean that the antigens are identical. It only represents serological identity or presence of common determinants.
Reaction of nonidentity is demonstrated by formation of precipitation lines crossing one another. Precipitin bands intersect each other, as they do not have common antigenic determinants.
Ouchterlony double diffusion technique is used for semiquantitative determinations of antigens or antibodies and to determine the purity and relatedness of the reactants.

110
Q

3 basic reaction patterns result from the relationship of antigen and antibody in Ouchterlony double diffusion technique. The appropriate observation for identity is:

1 Lines of precipitation run parallel to each other
2 Lines of precipitation merge with spur formation
3 Lines of precipitation coming together forming a smooth curve
4 Lines of precipitation cross each other
5 Lines of precipitation form 2 concentric circles

A

Lines of precipitation coming together forming a smooth curve

In Ouchterlony double diffusion technique, both antigen and antibody diffuse independently through the medium in vertical and horizontal dimensions. In this method, wells are cut into an agar plate and antigens and antisera are placed in the wells. The diffusion results in formation of a visible precipitation band at the zone of equivalence. When antibodies are placed in the center well and antigens are placed in the surrounding wells, 3 basic reactions are possible depending on the relationship between the antigens and antibody.
Reaction of partial identity is indicated when precipitation line fuses with spur formation. This reaction occurs when antigens being tested are nonidentical but do share common determinants.
Reaction of identity is revealed when the precipitation lines meet at a point and form a smooth curve or arc. This type of precipition band indicates that antibody is precipitating identical antigen specificities in each preparation. Lines of identity do not necessarily mean that the antigens are identical. It only represents serological identity or presence of common determinants.
Reaction of nonidentity is demonstrated by formation of precipitation lines crossing one another. Precipitin bands intersect each other, as they do not have common antigenic determinants.
Ouchterlony double diffusion technique is used for semiquantitative determinations of antigens or antibodies and to determine the purity and relatedness of the reactants.

111
Q

3 basic reaction patterns result from the relationship of antigen and antibody in Ouchterlony double diffusion technique. The appropriate observation for nonidentity is

1 Lines of precipitation run parallel to each other
2 Lines of precipitation merge with spur formation
3 Lines of precipitation coming together forming a smooth curve
4 Lines of precipitation cross each other
5 Lines of precipitation form 2 concentric circles

A

Lines of precipitation cross each other

In Ouchterlony double diffusion technique, both antigen and antibody diffuse independently through the medium in vertical and horizontal dimensions. In this method, wells are cut into an agar plate and antigens and antisera are placed in the wells. The diffusion results in formation of a visible precipitation band at the zone of equivalence. When antibodies are placed in the center well and antigens are placed in the surrounding wells, 3 basic reactions are possible depending on the relationship between the antigens and antibody.
Reaction of partial identity is indicated when precipitation line fuses with spur formation. This reaction occurs when antigens being tested are nonidentical but do share common determinants.
Reaction of identity is revealed when the precipitation lines meet at a point and form a smooth curve or arc. This type of precipition band indicates that antibody is precipitating identical antigen specificities in each preparation. Lines of identity do not necessarily mean that the antigens are identical. It only represents serological identity or presence of common determinants.
Reaction of nonidentity is demonstrated by formation of precipitation lines crossing one another. Precipitin bands intersect each other, as they do not have common antigenic determinants.
Ouchterlony double diffusion technique is used for semiquantitative determinations of antigens or antibodies and to determine the purity and relatedness of the reactants.

112
Q

What is the most correct statement concerning antigen antibody reaction?

1 Optimum precipitation occurs in area of antigen excess
2 Rocket immunoelectrophoresis is slower than radial immunodiffusion and can not be used for immunoglobulin quantitation
3 Immunoelectrophoresis procedure is used for identification of Bence Jones proteins in myeloma
4 Western blot test is an adaptation of passive agglutination technique
5 Nephelometry can not be used for quantitative measurements of immunoglobulins

A

Immunoelectrophoresis procedure is used for identification of Bence Jones proteins in myeloma

Immunoelectrophoresis procedure utilizes double diffusion and electrophoresis principles. Immunoelectrophoresis procedure is used for identification of free κ and λ chains in paraprotein disease. Using specific anti-κ and anti-λ antisera, monoclonal nature of Bence Jones proteins in myeloma can be determined.
The largest amount of precipitate is found in the zone of equivalence where the number of multivalent sites of antigen and antibody are approximately equal.
Rocket immunoelectrophoresis applies electrophoresis for facilitating migration of antigen in the agar. It is much more rapid than radial immunodiffusion (RID), and results can be obtained in a few hours. RID is modification of single diffusion method and takes more than 18 hours to obtain the results. Rocket immunoelectrophoresis is used for quantitative measurements of immunoglobulins, alpha-feto-protein in amniotic fluid, and complement components in body fluid.
Western blot test is an adaptation of immunofixation electrophoresis technique and is used as a confirmatory test to detect antibodies to HIV-1.
Nephelometry can be used for quantitative measurements of immunoglobulins, complement components, haptoglobin, C-reactive protein, and other serum proteins.

113
Q

Which of the following is true of prozone phenomenon?

1 Occurs in the zone of equivalence.
2 Occurs in the zone of antibody excess.
3 Occurs in zone of antigen excess.
4 Results in false positive reaction.
5 Results in maximum precipitation.

A

Occurs in the zone of antibody excess.

Formation of precipitation is highly dependent on relative concentration of antigen and antibody. Prozone phenomenon results from antibody excess and can lead to false negative reaction. In prozone phenomenon, antigen combines with only one or two antibody molecules failing to form a cross-linkage. When false negative reaction due to prozone phenomenon is suspected, the test may be repeated after dilution of antibody.
Excess of antigen leads to postzone phenomenon and gives false negative reaction.
The largest amount of precipitate is found in the zone of equivalence where the number of multivalent sites of antigen and antibody are approximately equal.

114
Q

Classical pathway is mediated by

1 Antigen antibody complex
2 Viral envelopes
3 Capsular polysaccharide
4 Endotoxins
5 Teichoic acids

A

Antigen antibody complex

Alternate pathway is activated by bacterial capsular polysaccharide, endotoxin, teichoic acids, and viral envelopes.
Classical pathway is mediated by antigen antibody complexes. The classes of immunoglobulin that can trigger classical pathway include IgM, IgG1, IgG2, and IgG3.
The complement proteins involved in classical pathway include C1q, C1r, C1s, C4, C2, C3, C5, C6, C7, C8, and C9. A single molecule of IgM or two cross-linked molecules of IgG are required to fix the complement. Antigen antibody complex binds to C1q subunit via receptor in Fc region of antibody and activates the complement cascade.

115
Q

The complement component/complex associated with bacterial cell lysis is

1 C1q
2 C3b
3 C4b2b
4 C5a
5 C5b6789

A

C5b6789

C5b6789 forms a membrane attack complex. Insertion of C5b6789 complex into cell membrane causes disruption of osmotic balance resulting in lysis of bacteria cells, red blood cells, and tumor cells.
C1 is composed of 3 subunits: C1q, C1r, and C1s. C1q is an aggregate of polypeptides that binds to receptor on Fc region of IgG or IgM, and classical pathway of the complement system is activated. C4b2b is C3 convertase and cleaves the C3 molecule into C3a and C3b. C3b is directly involved with opsonization of bacteria. C5a is a potent anaphylotoxin and a chemotactic factor.

116
Q

Hereditary angioedema is caused by deficiency of

1 C1q
2 C3
3 C1 inhibitor
4 Decay accelerating factor
5 Hagman factor

A

C1 inhibitor

C1 inhibitor or C1INH is a glycoprotein that binds to C1q causing dissociation of C1r and C1s which leads to inhibition of further enzymatic activity. C1INH also inhibits components required for clot formation such as activated Hagman factor, plasmin, kallikrein, and activated factor XI.
Hereditary angioedema is caused by deficiency of C1INH. It is inherited as an autosomal dominant gene that codes for dysfunctional protein or an inactive protein. One normal gene can not produce sufficient C1INH to keep up with the demand. Hereditary angioedema is characterized by recurrent attacks of angioedema, which effects the extremities, the skin, oropharynx, and other mucosal surfaces. Reduced levels of C4 and C2 and normal levels of C1q and C3 are found in hereditary angioedema. C1q and C3 levels are decreased in acquired C1INH deficiency.
Deficiency of C1INH leads to excess cleavage of C2 and C4 and perpetuation of classical pathway. This triggers increased production of kinin-related protein causing increased vascular permeability. Continued swelling and edema occurs in response to localized antigen stimulus.

117
Q

Which of the following is a B-cell differentiation factor that enhances IgA switching?

1 Interleukin-2
2 Interleukin-3
3 Interleukin-5
4 Interleukin-7
5 Interleukin-8
6 Interleukin-10
7 Interleukin-13

A

Interleukin-5

Interleukins are a group of cytokines that are polypeptide products of activated cells. Interleukins control a variety of cellular responses and contribute to regulation of immune response.
Lymphokines are primarily produced by activated T cells. Interleukin-2 (IL-2) is a T cell growth factor secreted by antigen-stimulated T helper cells. IL-2 causes proliferation and clonal expansion of activated T cells. IL-2 also enhances proliferation of B cells and IgG2 antibody production by B cells. IL-2 assists in clearing any foreign antigen from the area by enhancing cytotoxicity of natural killer cells (NK cells).
Interleukin-3 (IL-3) is a lymphokine secreted by activated T cells and is a major immunologic mediator of myeloid differentiation. IL-3 serves as an important link between the immune system and the hematopoietic system. IL-3 causes proliferation of granulocytes and macrophages. IL-3 is also associated with regulation and differentiation of megacaryocytic, erythrocytic, and mast cell precursors.
Interleukin-5 (IL-5), also known as B-cell differentiation factor, is produced by activated T helper cells and mast cells. IL-5 causes proliferation of activated B cells and increased eosinophil production. IL-5 also enhances IgM secretion and class switching of IgA.
Interleukin-7 (IL-7) is a hematopoietic growth factor secreted by bone marrow stromal cells. IL-7 functions as a growth factor for both T cell and B cell precursors. IL-7 also enhances cytotoxic activity of lymphocytes and monocytes.
Interleukin-8 (IL-8) is an inflammatory cytokine secreted by monocytes and macrophages. IL-8 is chemotactic for both neutrophils and T cells. IL-8 causes activation of neutrophils to release lysosomal enzymes and leads to superoxide dismutase production that is lethal for pathogens.
Interleukin-10 (IL-10) is produced by activated T cells and mast cells. IL-10 inhibits cytokine synthesis and T cell proliferation in the presence of monocytes. In particular, IL-10 inhibits interleukin-2 and interferon-γ production by T helper cells. IL-10 enhances B cell differentiation, leading to increased antibody response.
Interleukin-13 (IL-13) is produced by T helper cells. IL-13 is an important regulator of inflammatory response because it inhibits activity and release of inflammatory cytokines by macrophages.

118
Q

The cytokine associated with inhibitor of inflammatory cytokines secreted by macrophages is

1 Interleukin-2
2 Interleukin-3
3 Interleukin-5
4 Interleukin-7
5 Interleukin-8
6 Interleukin-10
7 Interleukin-13

A

Interleukin-13

Interleukins are a group of cytokines that are polypeptide products of activated cells. Interleukins control a variety of cellular responses and contribute to regulation of immune response.
Lymphokines are primarily produced by activated T cells. Interleukin-2 (IL-2) is a T cell growth factor secreted by antigen-stimulated T helper cells. IL-2 causes proliferation and clonal expansion of activated T cells. IL-2 also enhances proliferation of B cells and IgG2 antibody production by B cells. IL-2 assists in clearing any foreign antigen from the area by enhancing cytotoxicity of natural killer cells (NK cells).
Interleukin-3 (IL-3) is a lymphokine secreted by activated T cells and is a major immunologic mediator of myeloid differentiation. IL-3 serves as an important link between the immune system and the hematopoietic system. IL-3 causes proliferation of granulocytes and macrophages. IL-3 is also associated with regulation and differentiation of megacaryocytic, erythrocytic, and mast cell precursors.
Interleukin-5 (IL-5), also known as B-cell differentiation factor, is produced by activated T helper cells and mast cells. IL-5 causes proliferation of activated B cells and increased eosinophil production. IL-5 also enhances IgM secretion and class switching of IgA.
Interleukin-7 (IL-7) is a hematopoietic growth factor secreted by bone marrow stromal cells. IL-7 functions as a growth factor for both T cell and B cell precursors. IL-7 also enhances cytotoxic activity of lymphocytes and monocytes.
Interleukin-8 (IL-8) is an inflammatory cytokine secreted by monocytes and macrophages. IL-8 is chemotactic for both neutrophils and T cells. IL-8 causes activation of neutrophils to release lysosomal enzymes and leads to superoxide dismutase production that is lethal for pathogens.
Interleukin-10 (IL-10) is produced by activated T cells and mast cells. IL-10 inhibits cytokine synthesis and T cell proliferation in the presence of monocytes. In particular, IL-10 inhibits interleukin-2 and interferon-γ production by T helper cells. IL-10 enhances B cell differentiation, leading to increased antibody response.
Interleukin-13 (IL-13) is produced by T helper cells. IL-13 is an important regulator of inflammatory response because it inhibits activity and release of inflammatory cytokines by macrophages.

119
Q

The cytokine produced by activated T cells and responsible for differentiation of hematopoietic stem cells and mast cells is

1 Interleukin-2
2 Interleukin-3
3 Interleukin-5
4 Interleukin-7
5 Interleukin-8
6 Interleukin-10
7 Interleukin-13

A

Interleukin-3

Interleukins are a group of cytokines that are polypeptide products of activated cells. Interleukins control a variety of cellular responses and contribute to regulation of immune response.
Lymphokines are primarily produced by activated T cells. Interleukin-2 (IL-2) is a T cell growth factor secreted by antigen-stimulated T helper cells. IL-2 causes proliferation and clonal expansion of activated T cells. IL-2 also enhances proliferation of B cells and IgG2 antibody production by B cells. IL-2 assists in clearing any foreign antigen from the area by enhancing cytotoxicity of natural killer cells (NK cells).
Interleukin-3 (IL-3) is a lymphokine secreted by activated T cells and is a major immunologic mediator of myeloid differentiation. IL-3 serves as an important link between the immune system and the hematopoietic system. IL-3 causes proliferation of granulocytes and macrophages. IL-3 is also associated with regulation and differentiation of megacaryocytic, erythrocytic, and mast cell precursors.
Interleukin-5 (IL-5), also known as B-cell differentiation factor, is produced by activated T helper cells and mast cells. IL-5 causes proliferation of activated B cells and increased eosinophil production. IL-5 also enhances IgM secretion and class switching of IgA.
Interleukin-7 (IL-7) is a hematopoietic growth factor secreted by bone marrow stromal cells. IL-7 functions as a growth factor for both T cell and B cell precursors. IL-7 also enhances cytotoxic activity of lymphocytes and monocytes.
Interleukin-8 (IL-8) is an inflammatory cytokine secreted by monocytes and macrophages. IL-8 is chemotactic for both neutrophils and T cells. IL-8 causes activation of neutrophils to release lysosomal enzymes and leads to superoxide dismutase production that is lethal for pathogens.
Interleukin-10 (IL-10) is produced by activated T cells and mast cells. IL-10 inhibits cytokine synthesis and T cell proliferation in the presence of monocytes. In particular, IL-10 inhibits interleukin-2 and interferon-γ production by T helper cells. IL-10 enhances B cell differentiation, leading to increased antibody response.
Interleukin-13 (IL-13) is produced by T helper cells. IL-13 is an important regulator of inflammatory response because it inhibits activity and release of inflammatory cytokines by macrophages.

120
Q

The cytokine synthesis inhibitory factor that inhibits IL-2 production by T helper cells and promotes B cell differentiation and antibody production is:

1 Interleukin-2.
2 Interleukin-3.
3 Interleukin-5.
4 Interleukin-7.
5 Interleukin-8.
6 Interleukin-10
7 Interleukin-13

A

Interleukin-10

Interleukins are a group of cytokines that are polypeptide products of activated cells. Interleukins control a variety of cellular responses and contribute to regulation of immune response.
Lymphokines are primarily produced by activated T cells. Interleukin-2 (IL-2) is a T cell growth factor secreted by antigen-stimulated T helper cells. IL-2 causes proliferation and clonal expansion of activated T cells. IL-2 also enhances proliferation of B cells and IgG2 antibody production by B cells. IL-2 assists in clearing any foreign antigen from the area by enhancing cytotoxicity of natural killer cells (NK cells).
Interleukin-3 (IL-3) is a lymphokine secreted by activated T cells and is a major immunologic mediator of myeloid differentiation. IL-3 serves as an important link between the immune system and the hematopoietic system. IL-3 causes proliferation of granulocytes and macrophages. IL-3 is also associated with regulation and differentiation of megacaryocytic, erythrocytic, and mast cell precursors.
Interleukin-5 (IL-5), also known as B-cell differentiation factor, is produced by activated T helper cells and mast cells. IL-5 causes proliferation of activated B cells and increased eosinophil production. IL-5 also enhances IgM secretion and class switching of IgA.
Interleukin-7 (IL-7) is a hematopoietic growth factor secreted by bone marrow stromal cells. IL-7 functions as a growth factor for both T cell and B cell precursors. IL-7 also enhances cytotoxic activity of lymphocytes and monocytes.
Interleukin-8 (IL-8) is an inflammatory cytokine secreted by monocytes and macrophages. IL-8 is chemotactic for both neutrophils and T cells. IL-8 causes activation of neutrophils to release lysosomal enzymes and leads to superoxide dismutase production that is lethal for pathogens.
Interleukin-10 (IL-10) is produced by activated T cells and mast cells. IL-10 inhibits cytokine synthesis and T cell proliferation in the presence of monocytes. In particular, IL-10 inhibits interleukin-2 and interferon-γ production by T helper cells. IL-10 enhances B cell differentiation, leading to increased antibody response.
Interleukin-13 (IL-13) is produced by T helper cells. IL-13 is an important regulator of inflammatory response because it inhibits activity and release of inflammatory cytokines by macrophages.

121
Q

The cytokine associated with lymphokine produced by antigen-stimulated T helper cells, causes proliferation of activated T and B cells, and enhances toxicity of NK cells is

1 Interleukin-2
2 Interleukin-3
3 Interleukin-5
4 Interleukin-7
5 Interleukin-8
6 Interleukin-10
7 Interleukin-13

A

Interleukin-2

Interleukins are a group of cytokines that are polypeptide products of activated cells. Interleukins control a variety of cellular responses and contribute to regulation of immune response.
Lymphokines are primarily produced by activated T cells. Interleukin-2 (IL-2) is a T cell growth factor secreted by antigen-stimulated T helper cells. IL-2 causes proliferation and clonal expansion of activated T cells. IL-2 also enhances proliferation of B cells and IgG2 antibody production by B cells. IL-2 assists in clearing any foreign antigen from the area by enhancing cytotoxicity of natural killer cells (NK cells).
Interleukin-3 (IL-3) is a lymphokine secreted by activated T cells and is a major immunologic mediator of myeloid differentiation. IL-3 serves as an important link between the immune system and the hematopoietic system. IL-3 causes proliferation of granulocytes and macrophages. IL-3 is also associated with regulation and differentiation of megacaryocytic, erythrocytic, and mast cell precursors.
Interleukin-5 (IL-5), also known as B-cell differentiation factor, is produced by activated T helper cells and mast cells. IL-5 causes proliferation of activated B cells and increased eosinophil production. IL-5 also enhances IgM secretion and class switching of IgA.
Interleukin-7 (IL-7) is a hematopoietic growth factor secreted by bone marrow stromal cells. IL-7 functions as a growth factor for both T cell and B cell precursors. IL-7 also enhances cytotoxic activity of lymphocytes and monocytes.
Interleukin-8 (IL-8) is an inflammatory cytokine secreted by monocytes and macrophages. IL-8 is chemotactic for both neutrophils and T cells. IL-8 causes activation of neutrophils to release lysosomal enzymes and leads to superoxide dismutase production that is lethal for pathogens.
Interleukin-10 (IL-10) is produced by activated T cells and mast cells. IL-10 inhibits cytokine synthesis and T cell proliferation in the presence of monocytes. In particular, IL-10 inhibits interleukin-2 and interferon-γ production by T helper cells. IL-10 enhances B cell differentiation, leading to increased antibody response.
Interleukin-13 (IL-13) is produced by T helper cells. IL-13 is an important regulator of inflammatory response because it inhibits activity and release of inflammatory cytokines by macrophages.

122
Q

The cytokine associated with neutrophil and T cell chemoattractant produced by monocytes and macrophages is

1 Interleukin-2
2 Interleukin-3
3 Interleukin-5
4 Interleukin-7
5 Interleukin-8
6 Interleukin-10
7 Interleukin-13

A

Interleukin-8

Interleukins are a group of cytokines that are polypeptide products of activated cells. Interleukins control a variety of cellular responses and contribute to regulation of immune response.
Lymphokines are primarily produced by activated T cells. Interleukin-2 (IL-2) is a T cell growth factor secreted by antigen-stimulated T helper cells. IL-2 causes proliferation and clonal expansion of activated T cells. IL-2 also enhances proliferation of B cells and IgG2 antibody production by B cells. IL-2 assists in clearing any foreign antigen from the area by enhancing cytotoxicity of natural killer cells (NK cells).
Interleukin-3 (IL-3) is a lymphokine secreted by activated T cells and is a major immunologic mediator of myeloid differentiation. IL-3 serves as an important link between the immune system and the hematopoietic system. IL-3 causes proliferation of granulocytes and macrophages. IL-3 is also associated with regulation and differentiation of megacaryocytic, erythrocytic, and mast cell precursors.
Interleukin-5 (IL-5), also known as B-cell differentiation factor, is produced by activated T helper cells and mast cells. IL-5 causes proliferation of activated B cells and increased eosinophil production. IL-5 also enhances IgM secretion and class switching of IgA.
Interleukin-7 (IL-7) is a hematopoietic growth factor secreted by bone marrow stromal cells. IL-7 functions as a growth factor for both T cell and B cell precursors. IL-7 also enhances cytotoxic activity of lymphocytes and monocytes.
Interleukin-8 (IL-8) is an inflammatory cytokine secreted by monocytes and macrophages. IL-8 is chemotactic for both neutrophils and T cells. IL-8 causes activation of neutrophils to release lysosomal enzymes and leads to superoxide dismutase production that is lethal for pathogens.
Interleukin-10 (IL-10) is produced by activated T cells and mast cells. IL-10 inhibits cytokine synthesis and T cell proliferation in the presence of monocytes. In particular, IL-10 inhibits interleukin-2 and interferon-γ production by T helper cells. IL-10 enhances B cell differentiation, leading to increased antibody response.
Interleukin-13 (IL-13) is produced by T helper cells. IL-13 is an important regulator of inflammatory response because it inhibits activity and release of inflammatory cytokines by macrophages.

123
Q

The cytokine associated with hematopoietic growth factor secreted by bone marrow stromal cells and associated with proliferation of pre-B cells, pre-T cells, and mature T cells is:

1 Interleukin-2
2 Interleukin-3
3 Interleukin-5
4 Interleukin-7
5 Interleukin-8
6 Interleukin-10
7 Interleukin-13

A

Interleukin-7

Interleukins are a group of cytokines that are polypeptide products of activated cells. Interleukins control a variety of cellular responses and contribute to regulation of immune response.
Lymphokines are primarily produced by activated T cells. Interleukin-2 (IL-2) is a T cell growth factor secreted by antigen-stimulated T helper cells. IL-2 causes proliferation and clonal expansion of activated T cells. IL-2 also enhances proliferation of B cells and IgG2 antibody production by B cells. IL-2 assists in clearing any foreign antigen from the area by enhancing cytotoxicity of natural killer cells (NK cells).
Interleukin-3 (IL-3) is a lymphokine secreted by activated T cells and is a major immunologic mediator of myeloid differentiation. IL-3 serves as an important link between the immune system and the hematopoietic system. IL-3 causes proliferation of granulocytes and macrophages. IL-3 is also associated with regulation and differentiation of megacaryocytic, erythrocytic, and mast cell precursors.
Interleukin-5 (IL-5), also known as B-cell differentiation factor, is produced by activated T helper cells and mast cells. IL-5 causes proliferation of activated B cells and increased eosinophil production. IL-5 also enhances IgM secretion and class switching of IgA.
Interleukin-7 (IL-7) is a hematopoietic growth factor secreted by bone marrow stromal cells. IL-7 functions as a growth factor for both T cell and B cell precursors. IL-7 also enhances cytotoxic activity of lymphocytes and monocytes.
Interleukin-8 (IL-8) is an inflammatory cytokine secreted by monocytes and macrophages. IL-8 is chemotactic for both neutrophils and T cells. IL-8 causes activation of neutrophils to release lysosomal enzymes and leads to superoxide dismutase production that is lethal for pathogens.
Interleukin-10 (IL-10) is produced by activated T cells and mast cells. IL-10 inhibits cytokine synthesis and T cell proliferation in the presence of monocytes. In particular, IL-10 inhibits interleukin-2 and interferon-γ production by T helper cells. IL-10 enhances B cell differentiation, leading to increased antibody response.
Interleukin-13 (IL-13) is produced by T helper cells. IL-13 is an important regulator of inflammatory response because it inhibits activity and release of inflammatory cytokines by macrophages.

124
Q

Which of the following is true of cytokines?

1 Tumor necrosis factors stimulate T cell proliferation
2 Tumor necrosis factor-α is secreted by macrophages
3 Tumor necrosis factor-β is secreted by bone marrow stromal cells
4 Interferon-β has antiviral activity and inhibits MHC class I expression
5 Transforming growth factor-β enhances B cell proliferation and suppresses secretion of inflammatory mediators

A

Tumor necrosis factor-α is secreted by macrophages

Tumor necrosis factors (TNF) possess many of the same activity as IL-1. TNF differs from IL-1 in that TNF are not able to stimulate T cell proliferation and IL-1 has only limited ability to lyse tumor cells.
Tumor necrosis factor-α (TNF-α) is secreted by macrophages when stimulated by antigen. TNF-α is associated with production of hemorrhagic necrosis in tumors.
Tumor necrosis factor-β (TNF-β) is produced by T cells and B cells. TNF-β is responsible for killing tumor cells and virally infected cells.
Interferon-β inhibits synthesis of viral proteins by triggering the host cell to produce a second protein that interferes with the viral protein synthesis. Interferon-β is primarily secreted by fibroblasts and epithelial cells and enhances MHC class I expression.
Transforming growth factor-β (TGF-β) inhibits T and B cell proliferation. TGF-β is chemotactic for monocytes and induces secretion of inflammatory mediators.

125
Q

T cells

1 Are thymus independent lymphocytes
2 Are responsible for cellular immune response
3 Lack antigen receptors on the cell surface
4 Differentiates into plasma cells
5 Consist of IgM on their surface

A

Are responsible for cellular immune response

T cells are thymus dependent lymphocytes as they mature in thymus. B cells are thymus independent lymphocytes that develops in the bursa of Fabricus. IgM is present on the surface of B cells and not T cell. Also B cells are associated with antibody production. B cells differentiate into plasma cells and memory cells. Specific antigen receptors are present on the cell surface of T cells and are involved in cell mediated immunity. T cells are associated with production of cytokines, which include synthesis of IL-2, IL-4, IL-5, and gamma interferons.

126
Q

Which of the following is a T cell receptor responsible for sheep red blood cell rosetting?

1 CD2
2 CD3
3 CD4
4 CD8
5 CD19

A

CD2

Proteins found on cell surfaces can be used as markers to differentiate T cells and B cells. The maturation stages of the 2 types of cells can also be distinguished using these antigens or surface markers. CD (cluster of differentiation) classification is used as a reference in standardizing names of membrane proteins found on all human white cells.
CD2 is the earliest T-cell marker and is present on every peripheral T cell. It is also expressed by NK cells. CD2 is a receptor for sheep red blood cells and demonstrates rosetting phenomenon in which sheep red blood cells adhere to CD2 receptor, encircling the T cells and forming a daisy pattern.
Mature T cells bearing the CD4 receptor are known as helper or inducer cells. CD4+ cells recognize antigens in association with MHC class II product.
The population of matured T cells that express CD8 antigens are T-cytotoxic cells. These cells provide primary line of defense against changed self antigen, especially virally infected cells and tumor cells. Direct cell contact can result in destruction of target cells without antibody involvement.
CD3 serves as T cell antigen receptor and is involved in signal transduction.
CD19 is found on B cells and acts as a co-receptor that assists in enhancing antibody production in mature B cells.

127
Q

Which of the following is a surface marker present primarily on T-cytotoxic cells and T-suppressor cells?

1 CD2
2 CD3
3 CD4
4 CD8
5 CD19

A

CD8

Proteins found on cell surfaces can be used as markers to differentiate T cells and B cells. The maturation stages of the 2 types of cells can also be distinguished using these antigens or surface markers. CD (cluster of differentiation) classification is used as a reference in standardizing names of membrane proteins found on all human white cells.
CD2 is the earliest T-cell marker and is present on every peripheral T cell. It is also expressed by NK cells. CD2 is a receptor for sheep red blood cells and demonstrates rosetting phenomenon in which sheep red blood cells adhere to CD2 receptor, encircling the T cells and forming a daisy pattern.
Mature T cells bearing the CD4 receptor are known as helper or inducer cells. CD4+ cells recognize antigens in association with MHC class II product.
The population of matured T cells that express CD8 antigens are T-cytotoxic cells. These cells provide primary line of defense against changed self antigen, especially virally infected cells and tumor cells. Direct cell contact can result in destruction of target cells without antibody involvement.
CD3 serves as T cell antigen receptor and is involved in signal transduction.
CD19 is found on B cells and acts as a co-receptor that assists in enhancing antibody production in mature B cells.

128
Q

Which of the following is a surface marker associated with T cell antigen receptor and signal transduction?

1 CD2
2 CD3
3 CD4
4 CD8
5 CD19

A

CD3

Proteins found on cell surfaces can be used as markers to differentiate T cells and B cells. The maturation stages of the 2 types of cells can also be distinguished using these antigens or surface markers. CD (cluster of differentiation) classification is used as a reference in standardizing names of membrane proteins found on all human white cells.
CD2 is the earliest T-cell marker and is present on every peripheral T cell. It is also expressed by NK cells. CD2 is a receptor for sheep red blood cells and demonstrates rosetting phenomenon in which sheep red blood cells adhere to CD2 receptor, encircling the T cells and forming a daisy pattern.
Mature T cells bearing the CD4 receptor are known as helper or inducer cells. CD4+ cells recognize antigens in association with MHC class II product.
The population of matured T cells that express CD8 antigens are T-cytotoxic cells. These cells provide primary line of defense against changed self antigen, especially virally infected cells and tumor cells. Direct cell contact can result in destruction of target cells without antibody involvement.
CD3 serves as T cell antigen receptor and is involved in signal transduction.
CD19 is found on B cells and acts as a co-receptor that assists in enhancing antibody production in mature B cells.

129
Q

Which of the following is a surface marker that acts as B cell co-receptor?

1 CD2
2 CD3
3 CD4
4 CD8
5 CD19

A

CD19

Proteins found on cell surfaces can be used as markers to differentiate T cells and B cells. The maturation stages of the 2 types of cells can also be distinguished using these antigens or surface markers. CD (cluster of differentiation) classification is used as a reference in standardizing names of membrane proteins found on all human white cells.
CD2 is the earliest T-cell marker and is present on every peripheral T cell. It is also expressed by NK cells. CD2 is a receptor for sheep red blood cells and demonstrates rosetting phenomenon in which sheep red blood cells adhere to CD2 receptor, encircling the T cells and forming a daisy pattern.
Mature T cells bearing the CD4 receptor are known as helper or inducer cells. CD4+ cells recognize antigens in association with MHC class II product.
The population of matured T cells that express CD8 antigens are T-cytotoxic cells. These cells provide primary line of defense against changed self antigen, especially virally infected cells and tumor cells. Direct cell contact can result in destruction of target cells without antibody involvement.
CD3 serves as T cell antigen receptor and is involved in signal transduction.
CD19 is found on B cells and acts as a co-receptor that assists in enhancing antibody production in mature B cells.

130
Q

Which cells eliminate virally-infected cells and recognize the antigen in conjunction with MHC class I molecule?

1 Kupffer cells
2 Eosinophils
3 Plasma cells
4 Cytotoxic T cells
5 Helper T cells
6 Natural killer cells

A

Cytotoxic T cells

Cytotoxic T cells are CD8+ with MHC class I restriction. Direct cell contact can result in destruction of target cells without antibody involvement. Cytotoxic T cells provide the primary line of defense against changed self antigen, especially virally infected cells and tumor cells. Cytotoxic T cells recognize antigen in association with MHC class I molecule, which is expressed by all nucleated host cells.
Helper T cells are CD4+ cells and recognize the antigen in conjunction with MHC class II molecule. Helper T cells are required for antibody response to most antigens. These cells control proliferation, differentiation, and isotype switching of B cells.
Natural killer cells (NK cells) are large granular lymphocytes and do not express most of the recognizable surface marker of T or B cells. NK cells can mediate cytolytic reactions and kill target cells without prior exposure to antigen. Cytolytic activity of NK cells is nonphagocytic, extracellular, and not restricted by major histocompatibility complex (MHC). NK cells are also involved in antibody dependent cell mediated cytotoxicity (ADCC).
When B cells are stimulated by an antigen, they undergo differentiation and clonal expansion. Some of the transformed B cells eventually become antibody-secreting plasma cells. Plasma cells are fully differentiated lymphocytes with abundant cytoplasmic immunoglobulins. Antibody production is the primary function of plasma cells.
Kupffer cells are fixed macrophages found in liver. These cells are associated with phagocytosis and antigen presentation.
Eosinophils are homeostatic regulator of inflammation and also play a key role in the host defense against some parasites, like nematodes. Eosinophils secrete cytotoxic enzymes in their granules when attached to the surface of the parasite via IgE and damage them by way of oxidative mechanism.

131
Q

Which cells synthesize and secrete immunoglobulins upon antigen stimulation?

1 Kupffer cells
2 Eosinophils
3 Plasma cells
4 Cytotoxic T cells
5 Helper T cells
6 Natural killer cells

A

Plasma cells

Cytotoxic T cells are CD8+ with MHC class I restriction. Direct cell contact can result in destruction of target cells without antibody involvement. Cytotoxic T cells provide the primary line of defense against changed self antigen, especially virally infected cells and tumor cells. Cytotoxic T cells recognize antigen in association with MHC class I molecule, which is expressed by all nucleated host cells.
Helper T cells are CD4+ cells and recognize the antigen in conjunction with MHC class II molecule. Helper T cells are required for antibody response to most antigens. These cells control proliferation, differentiation, and isotype switching of B cells.
Natural killer cells (NK cells) are large granular lymphocytes and do not express most of the recognizable surface marker of T or B cells. NK cells can mediate cytolytic reactions and kill target cells without prior exposure to antigen. Cytolytic activity of NK cells is nonphagocytic, extracellular, and not restricted by major histocompatibility complex (MHC). NK cells are also involved in antibody dependent cell mediated cytotoxicity (ADCC).
When B cells are stimulated by an antigen, they undergo differentiation and clonal expansion. Some of the transformed B cells eventually become antibody-secreting plasma cells. Plasma cells are fully differentiated lymphocytes with abundant cytoplasmic immunoglobulins. Antibody production is the primary function of plasma cells.
Kupffer cells are fixed macrophages found in liver. These cells are associated with phagocytosis and antigen presentation.
Eosinophils are homeostatic regulator of inflammation and also play a key role in the host defense against some parasites, like nematodes. Eosinophils secrete cytotoxic enzymes in their granules when attached to the surface of the parasite via IgE and damage them by way of oxidative mechanism.

132
Q

Which cells are phagocytic antigen presenting cells found in liver cells

1 Kupffer cells
2 Eosinophils
3 Plasma cells
4 Cytotoxic T cells
5 Helper T cells
6 Natural killer cells

A

Kupffer cells

Cytotoxic T cells are CD8+ with MHC class I restriction. Direct cell contact can result in destruction of target cells without antibody involvement. Cytotoxic T cells provide the primary line of defense against changed self antigen, especially virally infected cells and tumor cells. Cytotoxic T cells recognize antigen in association with MHC class I molecule, which is expressed by all nucleated host cells.
Helper T cells are CD4+ cells and recognize the antigen in conjunction with MHC class II molecule. Helper T cells are required for antibody response to most antigens. These cells control proliferation, differentiation, and isotype switching of B cells.
Natural killer cells (NK cells) are large granular lymphocytes and do not express most of the recognizable surface marker of T or B cells. NK cells can mediate cytolytic reactions and kill target cells without prior exposure to antigen. Cytolytic activity of NK cells is nonphagocytic, extracellular, and not restricted by major histocompatibility complex (MHC). NK cells are also involved in antibody dependent cell mediated cytotoxicity (ADCC).
When B cells are stimulated by an antigen, they undergo differentiation and clonal expansion. Some of the transformed B cells eventually become antibody-secreting plasma cells. Plasma cells are fully differentiated lymphocytes with abundant cytoplasmic immunoglobulins. Antibody production is the primary function of plasma cells.
Kupffer cells are fixed macrophages found in liver. These cells are associated with phagocytosis and antigen presentation.
Eosinophils are homeostatic regulator of inflammation and also play a key role in the host defense against some parasites, like nematodes. Eosinophils secrete cytotoxic enzymes in their granules when attached to the surface of the parasite via IgE and damage them by way of oxidative mechanism.

133
Q

Which cells interact with the larval stage of some helminth parasites and damage them by way of oxidative mechanism?

1 Kupffer cells
2 Eosinophils
3 Plasma cells
4 Cytotoxic T cells
5 Helper T cells
6 Natural killer cells

A

Eosinophils

ytotoxic T cells are CD8+ with MHC class I restriction. Direct cell contact can result in destruction of target cells without antibody involvement. Cytotoxic T cells provide the primary line of defense against changed self antigen, especially virally infected cells and tumor cells. Cytotoxic T cells recognize antigen in association with MHC class I molecule, which is expressed by all nucleated host cells.
Helper T cells are CD4+ cells and recognize the antigen in conjunction with MHC class II molecule. Helper T cells are required for antibody response to most antigens. These cells control proliferation, differentiation, and isotype switching of B cells.
Natural killer cells (NK cells) are large granular lymphocytes and do not express most of the recognizable surface marker of T or B cells. NK cells can mediate cytolytic reactions and kill target cells without prior exposure to antigen. Cytolytic activity of NK cells is nonphagocytic, extracellular, and not restricted by major histocompatibility complex (MHC). NK cells are also involved in antibody dependent cell mediated cytotoxicity (ADCC).
When B cells are stimulated by an antigen, they undergo differentiation and clonal expansion. Some of the transformed B cells eventually become antibody-secreting plasma cells. Plasma cells are fully differentiated lymphocytes with abundant cytoplasmic immunoglobulins. Antibody production is the primary function of plasma cells.
Kupffer cells are fixed macrophages found in liver. These cells are associated with phagocytosis and antigen presentation.
Eosinophils are homeostatic regulator of inflammation and also play a key role in the host defense against some parasites, like nematodes. Eosinophils secrete cytotoxic enzymes in their granules when attached to the surface of the parasite via IgE and damage them by way of oxidative mechanism.

134
Q

The CD4+ cells involved in interaction with HLA class II antigen are

1 Kupffer cells
2 Eosinophils
3 Plasma cells
4 Cytotoxic T cells
5 Helper T cells
6 Natural killer cells

A

Helper T cells

Cytotoxic T cells are CD8+ with MHC class I restriction. Direct cell contact can result in destruction of target cells without antibody involvement. Cytotoxic T cells provide the primary line of defense against changed self antigen, especially virally infected cells and tumor cells. Cytotoxic T cells recognize antigen in association with MHC class I molecule, which is expressed by all nucleated host cells.
Helper T cells are CD4+ cells and recognize the antigen in conjunction with MHC class II molecule. Helper T cells are required for antibody response to most antigens. These cells control proliferation, differentiation, and isotype switching of B cells.
Natural killer cells (NK cells) are large granular lymphocytes and do not express most of the recognizable surface marker of T or B cells. NK cells can mediate cytolytic reactions and kill target cells without prior exposure to antigen. Cytolytic activity of NK cells is nonphagocytic, extracellular, and not restricted by major histocompatibility complex (MHC). NK cells are also involved in antibody dependent cell mediated cytotoxicity (ADCC).
When B cells are stimulated by an antigen, they undergo differentiation and clonal expansion. Some of the transformed B cells eventually become antibody-secreting plasma cells. Plasma cells are fully differentiated lymphocytes with abundant cytoplasmic immunoglobulins. Antibody production is the primary function of plasma cells.
Kupffer cells are fixed macrophages found in liver. These cells are associated with phagocytosis and antigen presentation.
Eosinophils are homeostatic regulator of inflammation and also play a key role in the host defense against some parasites, like nematodes. Eosinophils secrete cytotoxic enzymes in their granules when attached to the surface of the parasite via IgE and damage them by way of oxidative mechanism.

135
Q

Which of the following is true of type I hypersensitivity reaction

1 Associated with passive cutaneous anaphylaxis
2 Involves complement
3 Associated with increased production of IgG
4 Occurs after 24 hours of exposure to antigen
5 Serum sickness is type I hypersensitivity reaction

A

Associated with passive cutaneous anaphylaxis

Passive cutaneous anaphylaxis is Type I hypersensitivity reaction. This reaction is characterized by short time lag, usually seconds to minutes, between exposure to antigen and the onset of symptoms. This reaction does not involve complement. The mediators are released from mast cells and basophils. IgE production is elevated in this type of reaction.
Elevated synthesis of IgG is associated with type II and type III hypersensitivity reaction.
Serum sickness is an example of type III hypersensitivity reaction.

136
Q

The clinical condition of serum sickness is associated with what type of hypersensitivity reaction?

1 Type I hypersensitivity reaction
2 Type II hypersensitivity reaction
3 Type III hypersensitivity reaction
4 Type IV hypersensitivity reaction
5 Type V hypersensitivity reaction

A

Type III hypersensitivity reaction

Serum sickness is an immune complex reaction which is type III hypersensitivity reaction. Serum sickness develops from injection of heterologous serum protein within 1 or 2 weeks after the initial exposure. It is characterized by protracted interaction between antigen and antibody accompanied by formation of antigen-antibody complexes.
Hypersensitivity to penicillin is a common cause of type I hypersensitivity reaction. The metabolic product of penicillin acts as hapten and binds to a protein molecule and leads to antibody production and type I hypersensitivity reaction. On repeated administration of penicillin, the antibody can react with the hapten or the intact drug causing fever, rashes, and anaphylactic reaction.
Transfusion reaction is type II hypersensitivity reaction. It is a cytotoxic type of reaction in which an antigen present on cell surface combines with antibody in the host and initiates complement mediated lysis. ABO incompatibility is the most common cause of acute hemolytic transfusion reaction and occurs during or immediately after blood transfusion. Administration of erythrocytes in the presence of preexisting antibodies induce antigen-antibody reaction with an activation of complement, kinin, plasminogen, and coagulation system.
Mantoux test is used to demonstrate exposure to Mycobacterium tuberculosis.
Mantoux test is a delayed type of hypersensitivity reaction which is type IV hypersensitivity reaction. Development of erythema and the induration at the site of injection indicates previous exposure or sensitization with the antigen. This type of reaction starts hours or days after contact with the antigen and consists mainly of macrophages and helper T lymphocytes.

137
Q

A 42-year-old woman with uncomplicated pneumococcal pneumonia starts a 10 day course of penicillin V. On the ninth day, she appears jaundiced and has a very low hematocrit. Additional tests show that both her reticulocyte count and indirect bilirubin levels are elevated. She is subsequently diagnosed with drug-induced warm autoimmune hemolytic anemia. What type of hypersensitivity reaction is associated with autoimmune hemolytic anemia?

1 Type I hypersensitivity reaction
2 Type II hypersensitivity reaction
3 Type III hypersensitivity reaction
4 Type IV hypersensitivity reaction
5 Type V hypersensitivity reaction

A

Type II hypersensitivity reaction

Type II hypersensitivity reaction can present as autoimmune hemolytic anemia. In type II (cytotoxic) hypersensitivity reaction, IgM or IgG antibodies bind to cell surface antigens and initiate complement mediated lysis. This case involves antibodies against red blood cells. Other clinical problems associated with type II hypersensitivity include transfusion reactions, and hemolytic disease of the newborn.
Type I hypersensitivity reaction is incorrect. Type 1 (anaphylactic) hypersensitivity reaction involves IgE-mediated immediate hypersensitivity. The mechanism of damage involves mast cell degranulation induced by allergen cross-linked IgE. Type 1 hypersensitivity can present as systemic anaphylaxis, allergic rhinitis, bronchial asthma, atopic dermatitis and food allergies.
Type III hypersensitivity reaction is incorrect. Type III (immune complex) hypersensitivity involves the lodging of immune complexes in the walls of blood vessels, activation of complement, and infiltration of polymorphonuclear cells. Examples include: serum sickness, glomerulonephritis, rheumatoid arthritis, systemic lupus erythematosus, and hypersensitivity pneumonitis.
Type IV hypersensitivity reaction is incorrect. Type IV (delayed-type) hypersensitivity reaction is due to antigen-specific TH1 cells that activate tissue macrophages and stimulate a local inflammatory response over 12-72 hours. The Mantoux test used to demonstrate exposure to Mycobacterium tuberculosis is an example of a delayed type IV hypersensitivity reaction. Other examples include: contact dermatitis, granulomatous hypersensitivity and acute tissue graft rejection.
Type V hypersensitivity reaction is incorrect. A type V classification is rarely used in the US. The type V classification is more likely used as a subcategory of Type II dealing with receptor mediated autoimmune diseases. In these diseases, antibodies bind to cell surface receptors which either prevent the intended ligand from binding with the receptor or mimic the effect of the ligand impairing cell signaling. Clinical examples are autoimmune diseases such as Graves Disease and Myasthenia Gravis. This case is not associated with an autoimmune disease.

138
Q

Which of the following is true of infections caused by human immunodeficiency virus?

1 CD8 to CD4 ratio is reduced
2 Does not infect monocytes
3 P24 protein on the viral envelope binds to CD10 protein receptor on target cell
4 Causes functional inactivation and lysis of helper T cells
5 Can not be acquired via sexual contact and blood products

A

Causes functional inactivation and lysis of helper T cells

Human immunodeficiency virus (HIV) is associated acquired immunodeficiency syndrome (AIDS) in human. The virus primarily infects CD4 positive cells. GP120 protein on the viral envelope binds to the CD4 protein receptor on the target cells. These cells include helper T cells, monocytes, macrophages, microglial brain cells, and B cells. HIV can be transmitted via sexual contact and infected blood products. HIV infection results in functional inactivation and lysis of helper T cells. This leads to decrease in CD4+ cells and reduced CD4 to CD8 ratio (less than 1.0). AIDS patients are highly susceptible to opportunistic infection.

139
Q

Kidney transplants between identical sisters is an example of

1 Autograft
2 Allograft
3 Xenograft
4 Syngraft

A

Syngraft

Kidney transplant between genetically identical sisters is an example of syngraft. In this type of transplant, grafted tissue or organ contains similar antigenic determinants.
Autograft involves tissue transplant (such as skin or bone marrow) from one position to another in the same individual. This type of graft involves no difference in antigenic determinants.
Allograft is graft exchange between genetically different recipient and donor of the same species. Antigenic compatibility must be tested in allograft as the graft contains antigen not present in the recipient.
Xenograft is a graft exchange between recipient and donor of the different species and mostly involves extensive immunologic differences.

140
Q

The oncofetal antigen used as tumor marker in monitoring and diagnosing of germ cell tumor and liver tumor is

1 CA 15-3
2 CA 125
3 Carcinoembryonic antigen (CEA)
4 Alpha-fetoprotein (AFP)
5 Prostatic acid phosphatase

A

Alpha-fetoprotein (AFP)

Tumor marker is a characteristic of a neoplastic cell and can be measured in tissues and body fluids using immunochemical assays. Detection of tumor marker is helpful in finding location of tumor. Quantitative determination of particular tumor marker is useful in monitoring the patient’s response to therapy.
Alpha-fetoprotein (AFP) is an oncofetal antigen which is normally secreted by fetal liver cells and yolk sac cells. AFP is not present in detectable levels after birth in normal individuals. AFP is a tumor associated antigen which is re-expressed by rapidly dividing cells or neoplastic cells.
Elevated levels of AFP are associated with liver tumors and germ cell tumors. AFP is a reliable tumor marker for monitoring patient’s response to chemotherapy and radiation therapy.
CA 15-3 is a tumor marker used in detection of breast cancer. It is also used for monitoring therapy and recurrence of breast cancer.
CA 125 is an oncofetal antigen used in diagnosis and monitoring of ovarian cancer.
Carcinoembryonic antigen (CEA) is associated with various types of cancer. CEA is of diagnostic value in monitoring patients with colon cancer.
Prostatic acid phosphatase is an enzyme used for diagnosis of prostatic carcinoma.

141
Q

A hallmark of the humoral immune response is the phenomenon referred to as class switching. Class switching results in the production of secretion of different types of antibodies, which are specific to the same antigen. Which class of antibodies is usually produced first in response to the presence of a foreign antigen in the bloodstream?

1 IgA
2 IgM
3 IgG
4 IgD
5 IgE

A

IgM

The primary immune response to a foreign antigen is typically characterized by the production of IgM antibodies by B-cells. Antibodies of the IgG class function to protect the extravascular compartment; IgA antibodies are most often associated with the protection of mucosa; IgD plays a role in influencing the function of lymphocytes; and IgE participates in the response to parasitic infections and in the allergic response.

142
Q

An antigen is defined as any substance that binds to antibodies and induces an immune cell response or induction of antibodies. Antigens are composed of epitopes or antigenic determinants that are recognized by antibodies. Haptens are small molecules, often drugs, capable of binding to antibodies but are insufficient to generate an immunological response. Immunogens are small molecules that are capable of producing an immunological response. How do haptens differ from antigens?
Answer Choices

1 Antigens are composed of carbohydrates or proteins
2 Haptens bind to antibodies by a different mechanism as compared to antigens
3 Antigens produce an immunological
response and haptens do not
4 Haptens produce an immunological response when conjugated to a large molecule
5 Antigens and haptens are composed of different immunological structures

A

Haptens produce an immunological response when conjugated to a large molecule

Antigens differ from haptens in their ability to generate an immunological response. Both haptens and antigens are capable of binding to antibodies in a similar mechanism or way. Antigens and haptens bind to antibodies by noncovalent mechanisms including hydrophobic, ionic, or Van der Waals forces. Antigens are not restricted in structure to carbohydrates or proteins. Antigens and haptens are not distinguished as different immunological structures. Haptens are small in size, can bind to antibodies but are insufficient in producing an immunological response unless conjugated to a larger protein. For example, penicillin-an antibiotic drug, is a hapten that can link to serum proteins and then elicit an immunological response.

143
Q

In your pediatric practice, you have an eight year old male whose chief complaint is supposed atopy, as observed and diagnosed by the patient’s mother. You are called to the emergency department of your local hospital where your patient is being treated for non-specific symptoms of an “allergic reaction” to water (bottled, of course). “What are we going to do?” wails the mother. Abandoning all of your training on bedside manner, you proceed to inform the mother what the physical characteristics of an allergen are. Which of the following is a physical characteristic of an allergen?

1 Small size (< 70 kDa)
2 Soluble in lipid
3 Labile
4 Degradable
5 Simplicity of structure

A

Degradable

he larger the molecule in general, the more likely it is to be antigenic. Most antigens are greater than 70 kDa in size. If a small particle is indeed antigenic, it most likely involves a hapten, which increases the size of the particle by attaching to it.
Most antigens are proteins, and lipid solubility may inhibit antigenicity in some cases. In order for the body to respond to an antigen, it must bind to specific receptors on the surface of immune cells. Lipid solubility will interfere with this binding.
The antigen must be rigid in order for it to bind to IgE on the surface of mast cells. Also, rigidity helps for increased affinity of binding.
Antigens must be degradable, so that they can be processed. For presentation of antigens, the particle must be degraded and then expressed on surface receptors of the antigen-presenting cell.
The more complex the antigen, the more likely an immune response will be mounted to it. Simple or homogenous structures are poor antigens.

144
Q

You perform a kidney transplant on a 62-year-old male. Unfortunately, the lab mis-typed the organ and your patient begins to reject the graft within a week of receiving it. As your patient’s macrophages and cytotoxic T cells begin to reject the grafted organ, what cells specifically are they rejecting?

1 Cells of the loop of Henle
2 Capillary endothelial cells
3 Granular juxtaglomerular cells
4 Cells of Bowman’s capsule
5 Cells of the collecting tubules

A

Capillary endothelial cells

The cells of the loops of Henle are not directly exposed to blood, therefore the period of recognition and rejection by the host would be longer.
The first cells that the host’s immune system encounter, are those of the capillaries in which they travel. All of the immune response is directed against these cells. Microthrombi form in the capillary beds of the transplanted organ and it dies from hypoxia.
Granular juxtaglomerular cells are not directly exposed to blood, therefore the period of recognition and rejection by the host would be longer.
Cells of Bowman’s capsule are in close proximity to the blood, but the endothelial cells that compose the glomerulus are more accessible to cell-mediated immune response.
Cells of the collecting tubules are not directly exposed to blood, therefore the period of recognition and rejection by the host would be longer.

145
Q

A proud mother brings her healthy 6-week-old daughter into your office for her first “shots.” As your nurse administers the vaccine, you reflect upon what you know about vaccines and humoral immunity. You realize that this infant is naive to the antigen that is being introduced to her system, so her initial antibody response will be weak. You take comfort in that subsequent introductions of the antigen will produce a more protective immune response. What class of antibodies will be produced by your young patient against the vaccine?

1 IgG1
2 IgG2
3 IgG3
4 IgM
5 IgE

A

IgM

IgG1 is produced on subsequent exposure to a specific antigen. Under the influence of cytokines such as IL-4 and contact with T-cells presenting antigen, B lymphocytes will switch the class of the immunoglobulin being produced to IgG1.
Immunoglobulin G2 is often produced under the influence of IL-4 after the initial production of IgM.
Immunoglobulin G3 is the least stable immunoglobulin produced. Initially IgM is produced, but under both genetic and paracrine influence, IgG3 can be produced.
Under normal circumstances, IgM is produced during the initial response to an antigen. Subsequent exposure to the same antigen results in class switching by the B cells to produce IgG1, IgG2, or IgG3.
IgE is produced often after chronic exposure to antigen.

146
Q

A 4-year-old male presents to your general practice with severe immunodeficiency. Your work with your patient indicates that humoral immunity appears to be effective, while cell-mediated immunity is not effective. You are trying to determine if the child’s problem lies primarily with antigen presentation or with the CD4 cells themselves. Which of the following interleukins that are produced by macrophages stimulate T helper 1 cell activity?

1 IL-1
2 IL-6
3 IL-12
4 TNF-α
5 IL-4

A

IL-12

IL-1 stimulates T helper 2 cell activity. T helper 2 activity is required for effective humoral immunity.
IL-6 stimulates antibody production by B cells. This cytokine is thought to participate in the immune response involving memory B cells.
IL-12 stimulates Th1 cells to produce IL-2 and IFN-α. Both IL-2 and IFN-α are necessary for the development of normal cell-mediated immunity.
TNF–α is not an interleukin. A product of macrophages, as well as several other cells of the immune system, TNF, plays a significant role in the general regulation of both humoral and cell-mediated immunity.
IL-4 is produced by Th2 cells and targets humoral immunity.

147
Q

As a researcher for a biomedical research company, you are interested in the development of a new pharmaceutical that induces tolerance to transplanted tissues that does not require long-term immunosuppressive therapy. You are currently looking at methods by which tolerance can be induced in an adult. In which of the following scenarios will tolerance to a foreign antigen be induced in T cells?

1 Binding of antigen by the TCR, but lack of secondary signal mediated through CD 28
2 During development, binding of MHC expressed on thymocytes by TCR on developing T cells
3 Specific antigens such as pneumococcal polysaccharide block the TCR, preventing further activation
4 Repeated exposure to high concentrations of antigen will result in clonal exhaustion
5 Addition of an antigen with an adjuvant will induce tolerance

A

Binding of antigen by the TCR, but lack of secondary signal mediated through CD 28

Binding of TCR and CD 28 is required for activation of a T cell. If only TCR is bound, the T cell will not respond to the antigen. By binding TCR and blocking CD28, the T cells would not respond to the antigen used to bind TCR.
While recognition of MHC on thymocytes is essential for T cells to become tolerant, this tolerance is to self-antigens. Use of this method would have to be done in utero and would have little practical application, as there would be no way to predict which antigens would be needed.
Certain antigens such as pneumococcal antigen will induce tolerance in B cells, but have not been demonstrated to induce tolerance in T cells.
Repeated exposure to high concentrations of antigen will induce tolerance in B cells and have not been demonstrated to induce tolerance in T cells. This is used in inhalant allergy immunotherapy in hopes to reduce the antibodies bound to the surface of mast cells.
Addition of an adjuvant to an antigen is a way to break tolerance.

148
Q

In the waiting room of your emergency department are five individuals with various hypersensitivities. One has developed acute serum sickness from a plasma transfusion. Another has chronic serum sickness as a result of systemic lupus erythematosus. Another is a newborn that is jaundiced and having labored breathing. Another has apparently developed respiratory problems related to working in a cave. The final patient is complaining of a rash on her finger caused by the ring that her fiancé gave her. Which of the patients has Type II hypersensitivity?

1 Acute serum sickness
2 Chronic serum sickness (SLE patient)
3 Hemolytic disease of the newborn
4 Hypersensitivity pneumonitis
5 Metal allergies

A

Hemolytic disease of the newborn

Acute serum sickness is due to immune complexes that are deposited within the capillary beds. In a plasma transfusion, the foreign proteins are targeted and removed by formation of immune complexes. This is an example of Type III hypersensitivity.
Chronic serum sickness results when a low level of circulating immune complexes is depositing in capillary beds. This is an example of Type III hypersensitivity.
Type II hypersensitivity is usually mediated through antibodies and complement and is directed against erythrocytes. In hemolytic disease of the newborn, maternal IgG directed against the fetal red cell antigens (Rh) activate complement and destroy the circulating red cells.
Hypersensitivity pneumonitis is initiated by immune complex deposition within the walls of the pulmonary capillaries, possibly as a result of chronic exposure to inhalant antigens. This is an example of Type III hypersensitivity.
Metal allergies are classified as type IV hypersensitivity.

149
Q

You are presenting information about a vaccination program to a group of young families. One nervous mother expresses concern that vaccines will cause her child to reject all of the cells of her body. You attempt to reassure her that nothing of the sort will occur. Which of the following statements about processing of exogenous and endogenous antigen is correct?

1 Exogenous antigen is processed by B cells, macrophages, and dendritic cells and presented to T cells on MHC class I molecules
2 Endogenous antigen is processed by B cells, macrophages, and dendritic cells and presented to T cells on MHC class I molecules
3 Endogenous antigen is processed by body cells and presented to B cells on MHC class II molecules
4 Endogenous antigen is processed by body cells and presented to T cells on MHC class I molecules
5 Exogenous antigen is processed by body cells and presented to T cells on MHC class II molecules

A

Endogenous antigen is processed by body cells and presented to T cells on MHC class I molecules

Exogenous antigen is antigen taken up by antigen-presenting cells (macrophages, B cells, and dendritic cells) and presented on MHC class II molecules to T cells.
Endogenous antigen is not processed by B cells, but by cells of the body and presented to CD 8 T cells on MHC class I molecules. Expression of MHC class II is limited to cells of the immune system.
Expression of MHC class II is limited to cells of the immune system.
Endogenous antigen is processed by cells of the body and presented to CD 8 T cells on MHC class I molecules. Expression of MHC class II is limited to cells of the immune system.
Expression of MHC class II is limited to cells of the immune system.

150
Q

A 4-year-old female presents to your pediatric clinic with an extensive history of recurrent infections. You wonder if something in her environment could have caused her thymus to involute prematurely. You note that she should be able to mount an immune response against thymic independent antigens. Which of the following best describes thymic independent antigens?

1 Somatic antigens that are presented outside the thymus
2 Antigens that activate the immune
response without relying upon T cells
3 Antigens that are presented to T cells in sites other than the thymus
4 Antigens that induce the immune response in cells that mature within the thymus
5 Antigens that are protein in nature

A

Antigens that activate the immune response without relying upon T cells

Thymic independent antigens are generally bacterial in origin, not somatic.
Thymic independent antigens are antigens that stimulate an immune response without the presence of T helper cells. These are often bacterial carbohydrates such as pneumococcal polysaccharides.
Antigens are presented to T cells in several locations throughout the body, including lymph nodes. In the adult, the thymus no longer exists.
A portion of the T cells life may be spent inside the thymus, however, they may also mature in various sites throughout the body.
Thymic independent antigens are more often polysaccharide in nature, not protein.

151
Q

Your 17-year-old male AIDS patient presents to your clinic with pneumonia. You check his CD 4+ levels by flow cytometry. They are extremely low. Which of the following functions of the immune system is most likely to be functioning normally in your patient?

1 Activation of cytotoxic T cells
2 Activation of macrophages
3 Stimulation of B cells to produce
antibody
4 Presentation of antigen
5 Secretion of lymphokines

A

Presentation of antigen

Cytotoxic T cells require co-stimulation by lymphokines from CD 4+ cells in order to activate against the antigen presented to them by antigen presenting cells.
Macrophages require reinforcement from IL-2 and IFN-(from TH-1 cells which are CD 4+).
Production of antibody is dependent upon secretion of IL-4 by TH-2 cells, which are CD 4+.
Antigen presentation is carried out by B cells, macrophages, and dendritic cells. CD 4 + T cells are “helper” cells that serve to activate both humoral and cell-mediated immunity.
Several lymphokines are secreted by CD4+ cells. These are vital for both cellular and humoral immunity.

152
Q

A novel treatment for transplant rejection targets blocking activity of CD 8+ lymphocytes to new antigen. The theory is that old cell-mediated immunity will remain intact, but the response to the endothelial cells of the graft will not occur. If this treatment is successful, which following activities will function normally?

1 Lysis virally infected cells
2 Lysis tumor cells
3 Production of antibody by B cells
4 Suppression of inappropriate immune responses
5 Secretion of lymphokines

A

Production of antibody by B cells

CD 8 + lymphocytes use MHC class I molecules on cells of the body to check for the presence of endogenous antigens such as virally infected cells.
CD 8 + lymphocytes use MHC class I molecules on cells of the body to check for the presence of endogenous antigens such as tumor cells.
CD 8 + lymphocytes play no role in the stimulation of antibody production.
CD 8 + cells play a major role in avoidance of autoimmunity by either direct suppression or lysis of immune cells that respond to autoantigens.
These cells secrete lymphokines as part of the immune response.

153
Q

Your 17-year-old male AIDS patient presents to your clinic with pneumonia. You check his CD 4+ levels by flow cytometry. They are extremely low. You know that both cell-mediated and humoral immunity will be affected. Which of the following requirements for activation of a B cell should be unaffected by the infection?

1 CD 40 on the B cell
2 IL-2
3 Antigen presentation
4 IL-4
5 IL-5

A

CD 40 on the B cell

HIV infection does not affect B cell expression of CD40, although CD40L-bearing T helper cells will be in short supply.
IL-2 is secreted by the T helper and stimulates the B cells to proliferate and produce antibodies.
Antigen presentation will be a hinderance as up-regulation of macrophages and B cells requires input from CD 4+ T lymphocytes.
IL-4 is secreted by the T helper and stimulates the B cells to proliferate and produce antibodies.
IL-5 is secreted by the T helper and mediates the effects of IL-4.

154
Q

A 7-year-old male presents to your practice with an extensive history of atopy and bacterial infections. You draw blood and submit a sample for electrophoresis that identifies a high level of IgE. Additionally, chemotactic studies indicate that the patient’s neutrophils are unable to respond to chemotaxins. Identify the immunological disease that results in high levels of IgE, but inability of neutrophils to respond to chemotactic stimuli

1 Job’s syndrome
2 Chediak-Higashi syndrome
3 Wiskott-Aldrich syndrome
4 Fanconi’s syndrome
5 X-linked granulomatous disease

A

Job’s syndrome

Defects in antigen processing cells and innate immune response often lead to a wide variety of syndromes. Job’s syndrome is characterized by neutrophils that are unable to respond to chemotaxins.
Chediak-Higashi syndrome is a lethal, progressive autosomal recessive disorder associated with massive leukocytes inclusions, histiocytic infiltration of organs and pancytopenia.
Wiskott-Aldrich syndrome is an immunodeficiency syndrome transmitted as an X-linked recessive trait. It is characterized by poor antibody response to polysaccharide antigens and dysfunction of cell-mediated immunity.
Fanconi’s syndrome is characterized by pancytopenia.
X-linked granulomatous disease is an autosomal recessive trait that is characterized by proliferation of cellular immune components.

155
Q

You examine a 7-year-old female with a history of urticaria. Her mother reports that in some instances, she has had difficulty breathing, while other times she has also had diarrhea. Lab results suggest that she has elevated levels of C1 in her blood stream. Identify the immunological disease that results from a deficiency in C1 esterase inhibitor, resulting in the indirect generation of vasoactive kinins.

1 X-linked agammaglobulinemia
2 Hereditary angioedema
3 Common variable hypogammaglobulinemia
4 Adenosine deaminase and nucleotide phosphorylase deficiency
5 DiGeorge syndrome

A

Hereditary angioedema

X-linked agammaglobulinemia is an autosomal recessive disease that manifests itself by a congenital absence of immunoglobulins and a reduced number of B cells.
In hereditary angioedema, C1 esterase inhibitor is not produced, and excessive production of vasoactive kinin results. This causes angioedema. It is also known as traveling edema or giant urticaria, as it has a tendency to occur and resolve in multiple areas in the body, including the viscera.
Common variable hypogammaglobulinemia usually manifests itself in late childhood or early adulthood. It is characterized by hyperplasia of lymph centers, but a lack of plasma cells. B cells fail to differentiate after proliferation when exposed to antigen. It is thought that recurrent bacterial infections during young adulthood, in concert with exhaustion of plasma cells, leads to the loss of the ability to produce effective, long-term antibody protection.
In adenosine deaminase and nucleotide deficiency both B cell and T cell function is deficient. This deficiency is the most common cause of SCID.
DiGeorge syndrome is a T cell deficiency that is a result of thymic hypoplasia.

156
Q

Your patient, a 37-year-old Caucasian woman, presents to you with systemic lupus erythematosus (SLE). She is suffering from glomerulonephritis and rheumatoid disease. She has no history of recurrent bacterial infections, edema, or neisserial infections. As far as the complement system is concerned, which of the following is your patient most likely deficient?

1 C3
2 C4a and C4b
3 C1-INH
4 C6 and C7
5 C9

A

C4a and C4b

Deficiency in C3 may result in glomerulonephritis, but would also involve recurrent bacterial infections.
90% of individuals lacking C4a and C4b develop SLE. In their absence, C3 does not bind immune complexes. Lack of C3 binding of immune complex results in decreased clearance of the complexes from the blood and subsequent deposition throughout the body. This initiates some of the symptoms associated with SLE.
Lack of C1-INH results in hereditary angioedema.
Deficiencies in C6 and C7 result in recurrent neisserial infections.
Deficiency of C9 results in recurrent neisserial infection.

157
Q

A four-month-old female presents to your pediatric practice listless and jaundiced. She is dehydrated. Her mother relates that she has never been very vigorous. She also relates that recently, she has been soaking 6 diapers per day. You order blood work, but are suspicious that she will be anemic and uremic. Hemolytic-uremic syndrome is associated with which of the following?

1 Congenital deficiency in factor H
2 Congenital deficiency in CR1
3 Acquired deficiency in C1s
4 Overproduction of C1-INH
5 Overproduction of C4a and C4b

A

Congenital deficiency in factor H

Lack of factor H or factor I results in uncontrolled activation of the alternate pathway and subsequent depletion of C3. Patients suffer from red cell destruction and renal failure.
Congenital deficiency in CR1 would not allow for clearance of red cells and immune-complexes bound to the CR1 from the blood. Patients with SLE may have lower numbers of CR1 on red cells, allowing immune complexes to remain in circulation longer.
Acquired deficiency in C1s is not seen in infants and usually results in SLE-like syndrome.
Overproduction of C1-INH is not commonly seen. In general, this would result in diminished activation of complement.
Overproduction of C4a and C4b does not commonly occur.

158
Q

A patient comes into your office for a preoperative exam. As part of the exam you determine that she has type O blood. The genes responsible for giving rise to the ABO blood group types code for

1 Cell surface glycoproteins
2 Different forms of hemoglobin
3 Specific sugar transferases
4 Specific glycosidases
5 Specific immunoglobulins

A

Specific sugar transferases

The surface of the human erythrocyte has a large number of antigenic determinants that give rise to 21 independent blood group systems. The ABO and Lewis are the most widely studied of these. The ABO type reflects different terminal sugar residues on surface glycosphingolipids. The genetic variation is achieved through specific glycosyl transferases that are responsible for the synthesis of these saccharide determinants. Although the ABO and Lewis antigens are glycoproteins with differing saccharide components, the genes responsible for the ABO and Lewis phenotypes code for specific glycosyl transferases. The genes for A, B, and O are found on human chromosome 9. N-acetylgalactosamine glycosyltransferase is the product of the A gene, galactosyltransferase the product of the B gene, and the O gene codes for an inactive enzyme. The Lewis gene codes for a fucosyltransferase that adds fucose to the peripheral N-acetyl glucosamine residue in the glycoprotein precursor. As a result of the ABO glycosyltransferases, the A antigen has a terminal N-acetylglucosamine residue and the B antigen has a galactose and the O antigen has neither. The removal of the A or B immunodeterminant by glycosidases (hydrolytic enzymes that degrade glycoproteins by splitting glycosidic bonds) gives the O phenotype. While DNA and the various RNA’s require templates for replication and transcription, the synthesis of complex carbohydrates is not template directed. The sequence of saccharides is determined by the specificity of the glycosyl transferases.

159
Q

A patient comes into your office for a preoperative exam. As part of the exam you determine that she has type O blood. Which of the following antigens would you expect to find on her red blood cells?

1 no A or B antigens present
2 A and B antigens present
3 A antigen only is present
4 B antigen only is present
5 A, B, and O antigen present

A

no A or B antigens present

The surface of the human erythrocyte has a large number of antigenic determinants that give rise to 21 independent blood group systems. The ABO and Lewis are the most widely studied of these. The ABO type reflects different terminal sugar residues on surface glycosphingolipids. The genetic variation is achieved through specific glycosyl transferases that are responsible for the synthesis of these saccharide determinants. Although the ABO and Lewis antigens are glycoproteins with differing saccharide components, the genes responsible for the ABO and Lewis phenotypes code for specific glycosyl transferases. The genes for A, B, and O are found on human chromosome 9. N-acetylgalactosamine glycosyltransferase is the product of the A gene, galactosyltransferase the product of the B gene, and the O gene codes for an inactive enzyme. The Lewis gene codes for a fucosyltransferase that adds fucose to the peripheral N-acetyl glucosamine residue in the glycoprotein precursor. As a result of the ABO glycosyltransferases, the A antigen has a terminal N-acetylglucosamine residue and the B antigen has a galactose and the O antigen has neither. The removal of the A or B immunodeterminant by glycosidases (hydrolytic enzymes that degrade glycoproteins by splitting glycosidic bonds) gives the O phenotype. While DNA and the various RNA’s require templates for replication and transcription, the synthesis of complex carbohydrates is not template directed. The sequence of saccharides is determined by the specificity of the glycosyl transferases.

160
Q

You are interested in learning more about the genetic structure of immunoglobulins. You arrange to work in a research lab and discover that a significant difference occurs between the antibodies produced when a protein is injected into a rabbit and those produced by a cloned hybridoma cell. A major difference is that the antibodies produced from a

1 Rabbit are all directed towards the same epitope on the protein
2 Cloned hybridoma cell cannot be of the IgG class
3 Cloned hybridoma cell are of a single class and directed towards the same epitope in the protein
4 Rabbit are composed of many immunoglobulin classes, but all directed towards the same epitope in the protein
5 cloned hybridoma cell are of many immunoglobulin classes, but all directed towards the same epitope in the protein

A

Cloned hybridoma cell are of a single class and directed towards the same epitope in the protein

Monoclonal antibodies are antibodies produced by a cloned hybridoma cell in culture. Therefore, each antibody is identical and directed towards the same epitope of the antigen. An epitope is the antigenic determinant. A protein can have many epitopes. In contrast, polyclonal antibodies, such as those obtained after injection of the protein into a rabbit or a goat, are heterogeneous. These antibody molecules are produced by different B-lymphocytes in the animal in response to the antigen. This represents a mixture of different antibodies each of which recognizes different epitopes on the antigen.

161
Q

Refer to the diagram below. If a patient is unable to produce IgA isotype due to a selective IgA-chain deficiency, what region of the immunoglobulin is affected?

1 A
2 B
3 C
4 D
5 E

A

C

The correct response is C.

An immunoglobulin’s isotype is determined by the constant portion of the heavy chain gene, which on a protein level corresponds to all of the Fc and part of the Fab fragment.
Letter A demarcates the variable portion of the light chain protein.
Letter B demarcates the constant portion of the light chain protein.
Letter D demarcates the variable portion of the heavy chain protein.
Letter E demarcates the variable portion, or the idiotype, of the immunoglobulin.